Med-Surg

अब Quizwiz के साथ अपने होमवर्क और परीक्षाओं को एस करें!

A client who is HIV positive asks the nurse, "How will I know when I have AIDS?" Which response is best for the nurse to provide? "Diagnosis of AIDS is made when you have 2 positive ELISA test results." "Diagnosis is made when both the ELISA and the Western Blot tests are positive." "I can tell that you are afraid of being diagnosed with AIDS. Would you like for me to call your minister?" "AIDS is diagnosed when a specific opportunistic infection is found in an otherwise healthy individual."

"AIDS is diagnosed when a specific opportunistic infection is found in an otherwise healthy individual." AIDS is diagnosed when one of several processes defined by the CDC is present in an individual who is not otherwise immunosuppressed (PCP, candidacies, cryptococcus, cryptosporidiosis, Kaposi's sarcoma, CNS lymphomas) and/or a CD4+ T cell count less than 200 (normal count 1,000).

A client taking furosemide, reports difficulty sleeping. Which question is important for the nurse to ask the client? "What dose of medication are you taking?" "Are you eating foods rich in potassium?" "Have you lost weight recently?" "At what time do you take your medication?"

"At what time do you take your medication?" For a client taking a loop diuretic who complains of sleep issues, the nurse needs to first determine at what time of day the client takes the medication. Because of the diuretic effect of furosemide, clients should take the medication in the morning to prevent nocturia which may be the reason for the sleep difficulties.

A client who is receiving chemotherapy asks the nurse, "Why is so much of my hair falling out each day?" Which response by the nurse best explains the reason for alopecia? "Chemotherapy affects the cells of the body that grow rapidly, both normal and malignant." "Alopecia is a common side effect you will experience during long-term steroid therapy." "Your hair will grow back completely after your course of chemotherapy is completed." "The chemotherapy causes permanent alterations in your hair follicles that lead to hair loss."

"Chemotherapy affects the cells of the body that grow rapidly, both normal and malignant." The common adverse effects of chemotherapy (nausea, vomiting, alopecia, bone marrow depression) are due to chemotherapy's effect on the rapidly reproducing cells, both normal and malignant.

The nurse is providing discharge instructions to a client who has undergone a left orchiectomy for testicular cancer. Which statement indicates that the client understands his post-operative care and prognosis? "I should continue to perform testicular self-examination (TSE) monthly on my remaining testicle." "I should wear an athletic supporter and cup to prevent testicular cancer in my remaining testicle." "I should always use a condom because I am at increased risk for acquiring a sexually transmitted disease." "I should make sure my sons know how to perform TSE because they are at increased risk for this type of cancer."

"I should continue to perform testicular self-examination (TSE) monthly on my remaining testicle." Although testicular cancer protocols, such as surgery, radiation, or chemotherapy, focus on the primary site of testicular cancer, these treatments do not reduce the risk of testicular cancer in the remaining testicle, so early recognition is the best prevention. The client's understanding is reflected in the statement to perform monthly TSE for changes in size, shape, or consistency of the testis that may indicate early cancer.

A 20-year-old female client calls the nurse to report a lump she found in her breast. Which response is the best for the nurse to provide? "Check it again in one month, and if it is still there schedule an appointment." "Most lumps are benign, but it is always best to come in for an examination." "Try not to worry too much about it, because usually, most lumps are benign." "If you are in your menstrual period it is not a good time to check for lumps."

"Most lumps are benign, but it is always best to come in for an examination." The nurse advising the client to come in provides the best response because it addresses the client's anxiety most effectively and encourages prompt and immediate action for a potential problem.

A 57-year-old male client is scheduled to have a stress-thallium test the following morning and is NPO after midnight. At 0130, he is agitated because he cannot eat and is demanding food. Which response is best for the nurse to provide to this client? "I'm sorry sir, you have a prescription for nothing by mouth from midnight tonight." "I will let you have one cracker, but that is all you can have for the rest of tonight." "What did the healthcare provider tell you about the test you are having tomorrow?" "The test you are having tomorrow requires that you have nothing by mouth tonight."

"The test you are having tomorrow requires that you have nothing by mouth tonight." Being direct and explaining to the client that the test requires him to be NPO, is the most therapeutic statement because the nurse is responding to the client's question and providing him the reason why.

The registered nurse (RN) is caring for a young adult who is having an oral glucose tolerance tests (OGTT). Which laboratory result should the RN assess as a normal value for the two hour postprandial result? 140 mg/dl. 160 mg/dl. 180 mg/dl. 200 mg/dl.

140 mg/dl. The two hour postprandial level should be less 140 mg/dl for a young adult client.

The registered nurse (RN) recognizes which client group is at the greatest risk for developing a urinary tract infection (UTI)? (Rank from highest risk to lowest risk.) A. Older males. B. School-age female. C. Older females. D. Adolescent males.

1. Older females 2. School-aged female 3. Older males 4. Adolescent males Hypoestrogenism and alkalotic urine are other age-related factors put older women at the highest risk for UTIs. School age girls (6 to 12 years) are at risk for UTIs due to a higher prevalence to taking baths instead of showers, but these risks can be controlled in this population as well as hypoestrogenism and alkalotic urine. Older men are at risk due to possible obstruction of the bladder due to benign prostatic hypertrophy (BPH).

Which finding should the nurse identify as most significant for a client diagnosed with polycystic kidney disease (PKD)? Hematuria. 2 pounds weight gain. 3+ bacteria in urine. Steady, dull flank pain.

3+ bacteria in urine. Urinary tract infections (UTI) for a client with polycystic kidney disease (PKD) require prompt antibiotic therapy to prevent renal damage and scarring which may cause further progression of the disease so bacteria in the urine would be significant finding.

Which client should be further assessed for an ectopic pregnancy? A 24-year-old with shoulder and lower abdominal quadrant pain. A 33-year-old with intermittent lower abdominal cramping. A 20-year-old with fever and right lower abdominal colic. A 40-year-old with jaundice and right lower abdominal pain.

A 24-year-old with shoulder and lower abdominal quadrant pain. A 24-year-old with sudden onset of lower abdominal quadrant pain should be assessed for an ectopic pregnancy. The pain can also be referred to the shoulder and may be associated with vaginal bleeding.

Based on an analysis of the client's rhythm, atrial fibrillation, the nurse should prepare the client for which treatment protocol? Diuretic therapy. Pacemaker implantation. Anticoagulation therapy. Cardiac catheterization.

Anticoagulation therapy. The client is experiencing atrial fibrillation, and the nurse should prepare the client for anticoagulation therapy which should be prescribed before rhythm control therapies to prevent cardioembolic events which result from blood pooling in the fibrillating atria.

In assessing cancer risk, the nurse identifies which woman as being at greatest risk of developing breast cancer? A 35-year-old multipara who never breastfed. A 50-year-old whose mother had unilateral breast cancer. A 55-year-old whose mother-in-law had bilateral breast cancer. A 20-year-old whose menarche occurred at age 9.

A 50-year-old whose mother had unilateral breast cancer. The most predictive risk factors for development of breast cancer are over 40 years of age and a positive family history (occurrence in the immediate family, i.e., mother or sister). Other risk factors include nulliparity, no history of breastfeeding, early menarche and late menopause, but are not considered as predictive as a positive history of an immediate family member and over 40 years old.

Which client should the nurse assess first? A 27-year-old complaining of severe back pain. A 63-year-old complaining of foot and ankle pain. A 49-year-old with pancreatitis complaining of unrelenting abdominal pain. A 55-year-old newly admitted client complaining of jaw pain and indigestion.

A 55-year-old newly admitted client complaining of jaw pain and indigestion. The 55-year-old client should be assessed first to rule out cardiac involvement because jaw pain and indigestion are common descriptors of myocardial injury.

A male client is admitted after falling from his bed. The healthcare provider (HCP) tells the family that he has an incomplete fracture of the humerus. The family ask the RN what this means. Which explanation by the nurse accurately describes the client's fracture? Straight fracture line that is also a simple, closed fracture. Nondisplaced fracture line that wraps around the bone. A complete fracture that also punctures the skin. A fracture that bends or splinters part of the bone.

A fracture that bends or splinters part of the bone. An incomplete fracture occurs when part of the bone is splintered (broken) and it has not gone completely through the thickness of the bone.

A client taking a thiazide diuretic for the past six months has a serum potassium level of 3. The nurse anticipates which change in prescription for the client? The dosage of the diuretic will be decreased. The diuretic will be discontinued. A potassium supplement will be prescribed. The dosage of the diuretic will be increased.

A potassium supplement will be prescribed. A potassium level of 3 is considered low (normal is 3.5 to 5). Taking a thiazide diuretic often results in a loss of potassium, so a potassium supplement needs to be prescribed to restore a normal serum potassium level.

Which information should the nurse obtain when performing an initial assessment of a client who presents to the emergency department with a painful ankle injury? (Select all that apply.) A. Quality of the pain. B. Signs of inflammation. C. Ankle range of motion. D. Muscle strength testing. E. Visible deformities of the joint.

A, B, C, E Initial assessment of a joint injury is performed to determine the extent of the damage. The nurse's initial assessment of a painful ankle injury should include pain quality, the presence of deformities, evidence of inflammation, and range of motion.

The registered nurse (RN) is caring for a client with peptic ulcer disease (PUD). What assessment should the RN identify and document that is consistent with PUD? (Select all that apply). A. Hematemesis. B. Gastric pain on an empty stomach. C. Colic-like pain with fatty food ingestion. D. Intolerance of spicy foods. E. Diarrhea and stearrhea.

A, B, D Manifestations of PUD include hematemesis, gastric pain, and spicy food intolerance.

The nurse is preparing a client for orthopedic surgery on the left leg and completing a safety checklist before transport to the operating room. Which items should the nurse remove from the client? (Select all that apply.) A. Nail polish. B. Hearing aid. C. Wedding band. D. Left leg brace. E. Contact lenses. F. Partial dentures.

A, B, E, F The removal of nail polish provides a more accurate pulse oximetry readings and evaluation of capillary refill. Hearing aids, contact lenses, and partial dentures are removed to prevent damage, loss or misplacement, or injury during surgery. Ideally, give the client's significant other the contact lenses if they are not the disposable ones, hearing aids and partial dentures once placed in an appropriate labeled container to hold for safe keeping. If no significant other is not able to hold onto the items, then secured them in an appropriate and safe place.

The nurse is assessing a client admitted from the emergency department with gastrointestinal bleeding related to peptic ulcer disease (PUD). Which physiological factors can produce ulceration? (Select all that apply.) A. Vagal stimulation. B. An increased level of stress. C. Decreased duodenal inhibition. D. Hypersecretion of hydrochloric acid. E. An increased number of parietal cells.

A, C, D, E Hypersecretion of gastric juices and an increased number of parietal cells that stimulate secretion are most often the causes of ulceration. Vagal stimulation and decreased duodenal inhibition also increase the secretion of caustic fluids.

In preparing to administer intravenous albumin to a client following surgery, which are the priority nursing interventions? (Select all that apply.) A. Set the infusion pump to infuse the albumin within four hours. B. Compare the client's blood type with the label on the albumin. C. Assign a UAP to monitor blood pressure q15 minutes. D. Administer through a large gauge catheter. E. Monitor hemoglobin and hematocrit levels. F. Assess for increased bleeding after administration.

A, D, E, F Albumin should be infused within four hours because it does not contain any preservatives. Any fluid remaining after four hours should be discarded. A large gauge catheter allows for fast infusion rate, which may be necessary. Hemodilution may decrease hemoglobin (HgB) and hematocrit (HCT) levels, so the HgB and HCT levels should be monitored while monitoring for bleeding because of the increased blood volume and blood pressure.

Which findings are within expected parameters of a normal urinalysis for an older adult? (Select all that apply.) A. pH 6. B. Nitrate small. C. Protein small. D. Sugar negative. E. Bilirubin negative. F. Specific gravity 1.015.

A, D, E, F A pH of 6.0 is within the normal pH range for urine. Glucosuria and bilirubinuria are abnormal and should be negative upon urinalysis. Normal changes associated with aging include decreased creatinine clearance and decreased concentrating and diluting abilities which influence the normal range of urine specific gravity, 1.001 to 1.035. Although common health problems associated with aging include renal insufficiency, urinary incontinence, urinary tract infection, and enlarged prostate, these are indicative of pathology which should be treated.

When planning care for a client with right renal calculi, which nursing diagnosis has the highest priority? Acute pain related to movement of the stone. Impaired urinary elimination related to obstructed flow of urine. Risk for infection related to urinary stasis. Deficient knowledge related to need for prevention of recurrence of calculi.

Acute pain related to movement of the stone. The nursing diagnosis of the highest priority is "Acute pain related the the renal calculi's movement".

A client with diabetes mellitus is experiencing polyphagia. Which outcome statement is the priority for this client? Fluid and electrolyte balance. Prevention of water toxicity. Reduced glucose in the urine. Adequate cellular nourishment.

Adequate cellular nourishment. Diabetes mellitus Type 1 is characterized by hyperglycemia that precipitates glucosuria and polyuria (frequent urination), polydipsia (excessive thirst), and polyphagia (excessive hunger). Polyphagia is a consequence of cellular malnourishment when insulin deficiency prevents utilization of glucose into the cell for energy, so the outcome statement should include stabilization of adequate cellular nutrition which is done by providing the insulin supplement the client needs.

The home health nurse is assessing a client with terminal lung cancer who is receiving hospice care. Which activity should be assigned to the hospice practical nurse (PN)? Administer medications for pain relief, shortness of breath, and nausea. Clarify family members' feelings about the meaning of client behaviors and symptoms. Develop a plan of care after assessing the needs of the client and family. Teach family members to recognize restlessness and grimacing as signs of client discomfort.

Administer medications for pain relief, shortness of breath, and nausea. Hospice care provides symptom management and pain control during the dying process and enhances the quality of life for a client who is terminally ill. Administering medication and monitoring for therapeutic and adverse effects are within the scope of practice for the PN.

A client with heart failure is prescribed digoxin 0.125 mg PO. The client's apical heart rate is70 beats per minute, blood pressure is 125/75 mmHg, and respirations are 18 breaths per minute. Which action should the nurse implement next? Administer the medication. Inform the healthcare provider. Review the vital sign flowsheet. Reassess the apical heart rate.

Administer the medication. Obtaining the apical heart rate is a common parameter prior to administering digoxin, which may indicate early digoxin toxicity if the heart rate is less than 60 beats per minute, so the dose should be administered since the client is not demonstrating any signs of toxicity.

A female client taking oral contraceptives reports to the nurse that she is experiencing calf pain. Which action should the nurse implement? Determine if the client has also experienced breast tenderness and weight gain. Encourage the client to begin a regular, daily program of walking and exercise. Advise the client to notify the healthcare provider for immediate medical attention. Tell the client to stop taking the medication for a week to see if symptoms subside.

Advise the client to notify the healthcare provider for immediate medical attention. Calf pain is indicative of thrombophlebitis, a serious, life-threatening complication associated with the use of oral contraceptives which requires further assessment and possibly immediate medical intervention.

Two days postoperative, a male client reports aching pain in his left leg. The nurse assesses redness and warmth on the lower left calf. Which intervention would be most helpful to this client? Apply sequential compression devices (SCDs) bilaterally. Assess for a positive Homan's sign in each leg. Pad all bony prominences on the affected leg. Advise the client to remain in bed with the leg elevated.

Advise the client to remain in bed with the leg elevated. For a client exhibiting symptoms of deep vein thrombosis (DVT), a complication of immobility, the initial care includes bedrest and elevation of the extremity.

A female client is recently diagnosed with Sarcoidosis. The client tells the registered nurse (RN) that she does not understand why she has this. When teaching the client, the RN should include that sarcoidosis most commonly occurs with which ethnic group of women? African American women. Caucasian women. Asian women. Hispanic women.

African American women. Sarcoidosis, an autoimmune inflammatory disease affecting multiple organs and has shown familial tendency due to multiple genes that together increase the susceptibility of developing the disease. In research studies it occurs more commonly in African American women (10-80 out of 100,000); compare to Caucasian women of the United States (8 out of 100,000).

A client who has a chronic cough with blood-tinged sputum returns to the unit after a bronchoscopy. What nursing interventions should be implemented in the immediate post-procedural period? Keep the client on bed rest for eight hours. Check vital signs every 15 minutes for two hours. Allow the client nothing by mouth until the gag reflex returns. Encourage fluid intake to promote elimination of the contrast media.

Allow the client nothing by mouth until the gag reflex returns. The nasal pharynx and oral pharynx are anesthetized with local anesthetic spray prior to bronchoscopy, and the bronchoscope is coated with lidocaine gel to inhibit the gag reflex and prevent laryngeal spasm during insertion. The client should be NPO until the client's gag reflex returns to prevent aspiration from any oral intake or secretions.

The nurse is assisting a client out of bed for the first time after surgery. Which action should the nurse do first? Place a chair at a right angle to the bedside. Encourage deep breathing prior to standing. Help the client to sit and dangle legs on the side of the bed. Allow the client to sit with the bed in a high Fowler's position.

Allow the client to sit with the bed in a high Fowler's position The first step in assisting a client out of bed for the first time after surgery is to raise the head of the bed to a high Fowler's position, which allows venous return to compensate from lying flat and the vasodilation effects of perioperative drugs. This helps prevent the client from becoming light-headed and decreases the chance of a client fall.

The nurse is teaching a client diagnosed with peripheral arterial disease. Which genitourinary system complication should the nurse include in the teaching? Altered sexual response. Sterility. Urinary incontinence. Decreased pelvic muscle tone.

Altered sexual response. Peripheral arterial disease (PAD) is a cardiovascular condition characterized by narrowing of the arteries and reduced blood flow to the extremities. PAD is known to alter the blood flow to the male's penis and is associated with erectile dysfunction in men.

The nurse is performing an ophthalmoscopic examination on a hypertensive client. When assessing the client, which finding indicates the severity of hypertension? Opaque color of the sclera. Transparency of the cornea. Amount of retinal vessel damage that has occurred. Constriction and dilatation of the pupils.

Amount of retinal vessel damage that has occurred. Examination of the blood vessels of the retina reveal any damange to the retinal vessels. This is a significant indication about how much damage the client's high blood pressure has done to vessels throughout the body. Retinal damage indicates that hypertension is moderate to severe.

The registered nurse (RN) is caring for a client with acute pancreatitis and reviews the admission laboratory results. What laboratory value should the RN anticipate being elevated with this diagnosis? Triglycerides. Amylase. Creatinine. Uric acid.

Amylase An elevated amylase level is associated with acute pancreatitis.

A female client requests information about using the calendar method of contraception. Which assessment is most important for the nurse to obtain? Amount of weight gain or weight loss during the previous year. An accurate menstrual cycle diary for the past 6 to 12 months. Skin pigmentation and hair texture for evidence of hormonal changes. Previous birth-control methods and beliefs about the calendar method.

An accurate menstrual cycle diary for the past 6 to 12 months. The fertile period, which occurs 2 weeks prior to the onset of menses, is determined using an accurate record of the number of days of the menstrual cycles for the past 6 months, so it is most important to emphasize to the client that accuracy and being compliant in recording the menstrual diary is the basis of the calendar method.

The nurse is taking a history of a newly diagnosed Type 2 diabetic who is beginning treatment. Which subjective information is most important for the nurse to note? A history of obesity. An allergy to sulfa drugs. Cessation of smoking three years ago. Numbness in the soles of the feet.

An allergy to sulfa drugs. An allergy to sulfa drugs may make the client unable to use some of the most common antihyperglycemic agents (sulfonylureas). The nurse needs to highlight this allergy for the healthcare provider.

A client with chronic atrial fibrillation and a slow ventricular response is scheduled for surgical placement of a permanent pacemaker. The client asks the nurse how this device will help. How should the nurse explain the action of a synchronous pacemaker? Ventricular irritability is prevented by the constant rate setting of the pacemaker. Ectopic stimulus in the atria is suppressed by the device usurping depolarization. An impulse is fired every second to maintain a heart rate of 60 beats per minute. An electrical stimulus is discharged when no ventricular response is sensed.

An electrical stimulus is discharged when no ventricular response is sensed. The artificial cardiac pacemaker is an electronic device used to pace the heart when the normal conduction pathway is damaged or diseased, such as a symptomatic dysrhythmias like atrial fibrillation with a slow ventricular response. Pacing modes that are synchronous (impulse generated on demand or as needed according to the client's intrinsic rhythm) send an electrical signal from the pacemaker to the wall of the myocardium, stimulating it to contract when no ventricular depolarization is sensed.

The PET (positron emission tomography) scan is commonly used with oncology clients to provide for which diagnostic information? A description of inflammation, infection, and tumors. Continuous visualization of intracranial neoplasms. Imaging of tumors without exposure to radiation. An image that describes metastatic sites of cancer.

An image that describes metastatic sites of cancer. PET scans provide information regarding certain diseases of the heart (determination of tissue viability), brain (dementia, Parkinson's disease), and early detection of tumors and their metastasis.

During an interview with a client planning elective surgery, the client asks the nurse, "What is the advantage of having a preferred provider organization insurance plan?" Which response is best for the nurse to provide? Neither plan allows selections of healthcare providers or hospitals. There are fewer healthcare providers to choose from than in an HMO plan. An individual may select healthcare providers from outside of the PPO network. An individual can become a member of a PPO without belonging to a group.

An individual may select healthcare providers from outside of the PPO network. The financial implication of selecting a provider from outside of the network is the feature most relevant to the average consumer. The nurse must have knowledge about preferred provider organizations (PPOs), which provides the option for the consumer to select a Healthcare Provider (HCP) from within the PPO network (in-network) at a reduced cost versus a higher cost for selecting an out-of-network HCP.

The nurse directs an unlicensed assistive personnel (UAP) to obtain the vital signs for a client who returns to the unit after having a mastectomy for cancer. Which information should the nurse provide the UAP? Using a pillow, elevate the client's arm with an IV infusing on the operative side. Apply the blood pressure cuff to the arm on the nonoperative side. Position the arm on the operative side close to the body. Collect a fingerstick blood specimen from the arm on the operative side.

Apply the blood pressure cuff to the arm on the nonoperative side. Postoperatively, the nurse should instruct the UAP that blood pressure readings should be obtained from the arm on the nonoperative side to reduce the risk of injury due to compromised lymphatic drainage. The arm on the operative side of the mastectomy should be elevated on a pillow above the level of the right atrium to facilitate lymphatic drainage.

A client with a chronic infection of Hepatitis C virus (HCV) is scheduled for a liver biopsy. Which intervention should the nurse perform after the procedure? Progress activity as soon as possible. Assess for signs of bleeding and hypovolemia. Place the client in the left lateral position. Monitor blood pressure, pulse, and breathing every 4 hours.

Assess for signs of bleeding and hypovolemia. Assessment for signs of bleeding should be implemented because internal bleeding is the greatest risk following a liver biopsy. Having the client placed at right lateral position, not the left side, applies pressure at the biopsy site.

A client has been hospitalized with a femur fracture and is being treated with traction. Which action by the nurse is the priority when caring for this client? Assess neurovascular status. Change the client's position. Inspect the traction equipment. Review pain medication orders.

Assess neurovascular status. The use of traction for long bone fractures reduces the potential for damage to the surrounding tissues. Reports of increased pain may indicate circulatory compromise or tissue damage (compartment syndrome). Assessing the client's neurovascular status is the nurse's highest priority.

The nurse is caring for a client receiving tamoxifen for the treatment of breast cancer. Which action should the nurse include in the client's plan of care? Increase fluid intake. Monitor sodium chloride intake. Assist the client in coping with hot flashes. Encourage milk products to increase calcium intake.

Assist the client in coping with hot flashes. Tamoxifen, an estrogen receptor blocking agent, can cause hot flashes, so client education regarding menopausal-like symptoms should be included in the plan of care.

A client who had abdominal surgery two days ago has prescriptions for intravenous morphine sulfate 4 mg every 2 hours and a clear liquid diet. The client describes feeling distended and has sharp, cramping gas pains. What nursing intervention should be implemented? Obtain a prescription for a laxative. Withhold all oral fluid and food. Assist the client to ambulate in the hall. Administer the prescribed morphine sulfate.

Assist the client to ambulate in the hall. Postoperative abdominal distention is caused by decreased peristalsis as a result of handling the intestine during surgery, limited dietary intake before and after surgery, and anesthetic and analgesic agents. Peristalsis is stimulated, flatus is passed, and distention is minimized by implementing early and frequent ambulation.

During suctioning, a client with an uncuffed tracheostomy tube begins to cough violently and dislodges the tracheostomy tube. Which action should the nurse implement first? Notify the healthcare provider for reinsertion. Attempt to reinsert the tracheostomy tube. Position the client in a lateral position with the neck extended. Ventilate client's tracheostomy stoma with a manual bag-mask.

Attempt to reinsert the tracheostomy tube. The nurse should attempt to reinsert the tracheostomy tube by using a hemostat to open the tracheostomy or by grasping the retention sutures (if present) to spread the opening and insert a replacement tube (with its obturator) into the stoma. Once in place, the obturator should immediately be removed.

A client arrives at the emergency department for treatment of injuries sustained in a motor vehicle collision. The nurse notes asymmetrical expansion of the chest wall during respiration. Which action should the nurse implement next? Auscultate the lungs bilaterally. Inspect the overall skin color. Palpate for tactile fremitus. Percuss thechest for resonance.

Auscultate the lungs bilaterally. Chest trauma may result in the development of pneumothorax. After noting asymmetric expansion of the chest wall, the nurse should auscultate the lungs to determine if the client can move air through all of the lung fields.

The nurse is preparing a teaching plan for a client with newly diagnosed glacoma and a history of allergic rhinitis. Which information is most important for the nurse to provide the client about using over-the-counter (OTC) medications for allergies? Notify your healthcare provider if there is an increase in heart rate. Increase fluid intake while taking an antihistamine or decongestant. Avoid allergy medications that contain pseudoephedrine or phenylephrine. Ophthalmic lubricating drops may be used for eye dryness due to allergy medications.

Avoid allergy medications that contain pseudoephedrine or phenylephrine. OTC allergy medications may contain ephedrine, phenylephrine, or pseudoephedrine, which can cause adrenergic side effects, such as increased intraocular pressure, so a client with glaucoma should avoid using these OTC medications.

A man who smokes two packs of cigarettes a day wants to know if smoking is contributing to the difficulty that he and his wife are having getting pregnant. What information is best for the nurse to provide? (Select all that apply.) A. Only marijuana cigarettes affect sperm count. B. Smoking can decrease the quantity and quality of sperm. C. The first semen analysis should be repeated to confirm sperm counts. D. Cessation of smoking improves general health and fertility. E. Sperm specimens should be collected in 2 subsequent days.

B, C, D The use of tobacco, alcohol, and marijuana may affect a man's sperm counts.

A male client who smokes two packs of cigarettes a day states he understands that smoking cigarettes is contributing to the difficulty that he and his wife are having in getting pregnant and wants to know if other factors could be contributing to their difficulty. What information is best for the nurse to provide? (Select all that apply.) A. Marijuana cigarettes do not affect sperm count. B. Alcohol consumption can cause erectile dysfunction. C. Low testosterone levels affect sperm production. D. Cessation of smoking improves general health and fertility. E. Obesity has no effect on sperm production.

B, C, D Use of tobacco, alcohol, and marijuana may affect sperm counts. Sperm count is also negatively affected by low testerone levels and obesity.

The nurse is providing postoperative instructions for a female client after a mastectomy. Which information should the nurse include in the teaching plan? (Select all that apply.) A. Empty surgical drains once a week using procedure gloves. B. Report inflammation of the incision site or the affected arm. C. Wear clothing with snug sleeves over the arm on the operative side. D. Avoid lifting more than 4.5 kg (10 lb) or reaching above her head.

B, D Part of a client's s/p mastectomy teaching plan should include reporting evidence of inflammation at the incision site or the affected arm, and to avoid lifting or reaching above their head.

A client presents with chronic venous insufficiency. Which assessment finding should the nurse anticipate? Bilateral lower leg stasis dermatitis. Clubbing of fingers and toes. Intermittent claudication. Incorrect Peripheral cyanosis.

Bilateral lower leg stasis dermatitis. Clients who suffer from chroninc venous insufficiency often develop statsis dermatitis in the lower extremities. Statis dermatitis appear as brownish-red discoloration on the lower extremities at the ankles which can develop into stasis ulcers due to the pooling of the venous blood flow back to the heart.

A client presents with chronic venous insufficiency. Which assessment finding should the nurse anticipate? Bilateral lower leg stasis dermatitis. Clubbing of fingers and toes. Intermittent claudication. Peripheral cyanosis.

Bilateral lower leg stasis dermatitis. Clients who suffer from chroninc venous insufficiency often develop statsis dermatitis in the lower extremities. Statis dermatitis appear as brownish-red discoloration on the lower extremities at the ankles which can develop into stasis ulcers due to the pooling of the venous blood flow back to the heart.

A client is admitted after blunt abdominal injury. Which assessment finding requires immediate action by the nurse? Radiating abdominal pain with left lower quadrant palpation. Grimacing after palpation of the right hypochondriac region. Rebound tenderness with abdominal palpation. Bluish periumbilical skin discoloration.

Bluish periumbilical skin discoloration. Immediate action is indicated for intraperitoneal hemorrhage which causes periumbilical discoloration and indicates the presence of a splenic rupture, a life-threatening complication of blunt abdominal injury.

What types of medications should the nurse expect to administer to a client during an acute respiratory distress episode? Vasodilators and hormones. Analgesics and sedatives. Anticoagulants and expectorants. Bronchodilators and steroids.

Bronchodilators and steroids. Besides supplemental oxygen, a client with acute respiratory distress syndrome (ARDS) needs medications to widen air passages, increase air space, and reduce alveolar membrane inflammation, such as bronchodilators and steroids.

A 58-year-old client who has been post-menopausal for five years is concerned about the risk for osteoporosis because her mother has the condition. Which information should the nurse offer? Osteoporosis is a progressive genetic disease with no effective treatment. Calcium loss from bones can be slowed by increasing calcium intake and exercise. Estrogen replacement therapy should be started to prevent the progression osteoporosis. Low-dose corticosteroid treatment effectively halts the course of osteoporosis.

Calcium loss from bones can be slowed by increasing calcium intake and exercise Post-menopausal females are at risk for osteoporosis due to the cessation of estrogen secretion, but a regimen including calcium, vitamin D, and weight-bearing exercise can help prevent further bone loss.

The nurse notes that the only ECG for a 55-year-old male client scheduled for surgery in two hours is dated two years ago. The client reports that he has a history of "heart trouble," but has no problems at present. Hospital protocol requires that those over 50 years of age have a recent ECG prior to surgery. Which nursing action is best for the nurse to implement? Ask the client what he means by "heart trouble." Call for an ECG to be performed immediately. Notify surgery that the ECG is over two years old. Notify the client's surgeon immediately.

Call for an ECG to be performed immediately. According to the hospital policy, clients over the age of 50 and/or with a history of cardiovascular disease, should receive ECG evaluation prior to surgery, generally 24 hours to two weeks before. The nurse needs to first arrange for an ECG to be performed immediately prior to surgery.

Which instruction should the nurse include in the discharge teaching for a client who needs to perform self-catheterization technique at home? Catheterize every 3 to 4 hours. Maintain sterile technique. Use the Cred maneuver before catheterization. Drink 500 mL of fluid within 2 hours of catheterization.

Catheterize every 3 to 4 hours. The average interval between catheterizations for adults is every 3 to 4 hours. Although sterile technique is indicated in healthcare facilities, clean technique is often followed by the client when performing self-catheterization at home.

A client with history of atrial fibrillation is admitted to the telemetry unit with sudden onset of shortness of breath. The nurse observes a new irregular heart rhythm and should perform which assessment at this time? Check for a pulse deficit. Palpate the apical impulse. Inspect jugular vein pulse. Examine for a carotid bruit.

Check for a pulse deficit. A client with a past history of atrial fibrillation may return to that rhythm. Any signs of atrial fibrillation, such as sudden onset shortness of breath, requires further investigation. The nurse should assess this client for a pulse deficit because this condition occurs with atrial fibrillation.

The nurse is caring for a client with a continuous feeding through a percutaneous endoscopic gastrostomy (PEG) tube. Which intervention should the nurse include in the plan of care? Flush the tube with 50 ml of water q 8 hours. Check for tube placement and residual volume q4 hours. Obtain a daily x- ray to verify tube placement. Position on left side with head of bed elevated 45 degrees.

Check for tube placement and residual volume q4 hours. Percutaneous endoscopic gastrostomy (PEG) tube placement and residual volume should be checked every four hours for clients on continuous feeding. If the gastric residual is more than 200mL for an adult client; stop the feeding and re-check the gastric residual one hour later. If the residual still remains more than 200mL; continue to keep the feeding on hold and contact the client's health care provider.

The nurse is caring for a client who is receiving chemotherapy for non-Hodgkin's lymphoma. Laboratory results reveal a platelet count of 10,000/mL. Which action should the nurse implement? Encourage fluids to 3,000 mL/day. Check stools for occult blood. Provide oral hygiene every 2 hours. Check for fever every 4 hours.

Check stools for occult blood. Platelet counts less than 100,000/mm3 are indicative of thrombocytopenia, a common side effect of chemotherapy. A client with thrombocytopenia should be assessed frequently for occult bleeding in the emesis, sputum, feces, urine, nasogastric secretions, or wounds.

A client who is fully awake after a gastroscopy asks the nurse for something to drink. After confirming that liquids are allowed, which assessment action should the nurse consider a priority? Listen to bilateral lung and bowel sounds. Obtain the client's pulse and blood pressure. Assist the client to the bathroom to void. Check the client's gag and swallow reflexes.

Check the client's gag and swallow reflexes. Following gastroscopy, a client should remain nothing by mouth until the effects of local anesthesia have dissipated and the airway's protective gag and swallow reflexes have returned.

A client who is admitted to the emergency department with a possible tension pneumothorax after a motor vehicle collision is having multiple diagnostic tests. Which finding requires immediate action by the nurse? Serum amylase of 132 units/L. Serum sodium of 134 mEq/L. Chest x-ray indicating a mediastinal shift. Abdominal x-ray air throughout intestines.

Chest x-ray indicating a mediastinal shift. Immediate action is required for findings of a mediastinal shift, which can precipitate life-threatening cardiovascular collapse as the great cardiac vessels become kinked and compressed due to the tension pneumothorax.

A female client admitted with abdominal pain is diagnosed with cholelithiasis. The client asks the registered nurse (RN) what she should expect as a common treatment. What recommended plan of care should the nurse provide the client? Rest with liquid diet only. Drugs such as ursodiol. Cholecystectomy via laparoscopy. LaVeen vena caval shunt.

Cholecystectomy via laparoscopy. The nurse should explain to the client that gall bladder surgical removal is most often recommended via laparoscopic excision.

A client is newly diagnosed with diverticulosis. The registered nurse (RN) is assessing the client's basic knowledge about the disease process. Which statement by the client conveys an understanding of the etiology of diverticula? Over use of laxatives for bowel regularity result in loss of peristaltic tone. Inflammation of the colon mucosa cause growths that protrude into the colon lumen. Diverticulosis is the result of high fiber diet and sedentary life style. Chronic constipation causes weakening of colon wall which result in out-pouching sacs.

Chronic constipation causes weakening of colon wall which result in out-pouching sacs. A client who has chronic constipation often strains to pass constipated stool which increases intestinal pressure that weakens the intestinal walls and causes out-pouching sacs, called diverticula which commonly occur in the sigmoid.

A male client comes into the clinic with a history of penile discharge with painful, burning unrination. Which action should the nurse implement? Collect a culture of the penile discharge. Palpate the inguinal lymph nodes gently. Observe for scrotal swelling and redness. Express the discharge to determine color.

Collect a culture of the penile discharge. Penile dischargewith painful urination is commonly associated with gonorrhea. The nurse should collect a culture of the penile discharge todetermine thecause of these symptoms. The cause must be determined or confirmed through culture to identify the organism and ensure effective treatment.

An older adult female client is brought to the clinic by her daughter for a flu shot. She has lost significant weight since the last visit. She has poor personal hygiene and inadequate clothing for the weather. The client states that she lives alone and denies problems or concerns. Which action should the nurse implement? Notify social services immediately of suspected elderly abuse. Discuss the need for mental health counseling with the daughter. Explain to the client that she needs to take better care of herself. Collect further data to determine whether self-neglect is occurring.

Collect further data to determine whether self-neglect is occurring. Changes in weight and hygiene may be indicators of self-neglect or neglect by family members. Further assessment is needed before notifying social services or discussing a need for counseling.

Which milestone indicates to the nurse successful achievement of young adulthood? Demonstrates a conceptualization of death and dying. Completes education and becomes self-supporting. Creates a new definition of self and roles with others. Develops a strong need for parental support and approval.

Completes education and becomes self-supporting. Transitioning through young adulthood is characterized by establishing independence as an adult, and includes developmental tasks such as completing education, beginning a career, and becoming self-supporting.

The nurse is assessing a client who is bedfast and refuses to turn or move from a supine position. How should the nurse assess the client for possible dependent edema? Compress the flank and upper buttocks. Measure the client's abdominal girth. Gently palpate the lower abdomen. Apply light pressure over the shins.

Compress the flank and upper buttocks. Dependent edema collects in dependent areas, such as the flank and upper buttocks of the client who is persistently flat in bed. By compressing these areas, the nurse can determine if any pitting edema is present.

An elderly client is admitted with a diagnosis of bacterial pneumonia. When observing the client for the first signs of decreasing oxygenation, the nurse should assess for which clinical cues? Abominal distention. Undue fatigue. Cyanosis of the lips. Confusion and tachycardia

Confusion and tachycardia The onset of pneumonia in the elderly may be signaled by general deterioration, confusion, increased heart rate or increased respiratory rate due to the decreased oxygen- carbon dioxide exchange at the alveoli, known as the V-Q mismatch. Cyanosis is a very late sign.

The nurse is teaching a client about precautions for a new prescription for lovastatin. Which symptom should the nurse instruct the client to report to the healthcare provider immediately? Terrible nightmares. Increased nocturia. Severe muscle pain. Visual disturbances.

Severe muscle pain. A potential, serious side effect of statin therapy that is used to lower both LDL-C and triglyceride levels is rhabdomyolysis, which is manifested by severe muscle pain and aching.

Which assessment finding is of greatest concern to the nurse who is caring for a client with stomatitis? Cough brought on by swallowing. Sore throat caused by speaking. Painful and dry oral cavity. Unintended weight loss.

Cough brought on by swallowing. A cough brought on by swallowing is a sign of dysphagia, which is a finding of particular concern in a client with stomatitis. Dysphagia can cause numerous problems, including airway obstruction, and should be reported to the healthcare provider immediately.

The nurse is assessing a client with bacterial meningitis. Which assessment finding indicates the client may have developed septic emboli? Cyanosis of the fingertips. Bradycardia and bradypnea. Presence of S3 and S4 heart sounds. 3+ pitting edema of the lower extremities.

Cyanosis of the fingertips. Septic emboli secondary to meningitis commonly lodge in the small arterioles of the extremities, causing a decrease in circulation to the hands which may lead to gangrene.

The nurse is teaching a female client who uses a contraceptive diaphragm about reducing the risk for toxic shock syndrome (TSS). Which information should the nurse include? (Select all that apply.) A. Remove the diaphragm immediately after intercourse. Incorrect B. Wash the diaphragm with an alcohol solution. C. Use the diaphragm to prevent conception during the menstrual cycle. D. Do not leave the diaphragm in place longer than 8 hours after intercourse. Correct E. Replace the old diaphragm every 3 months.

D, E The diaphragm needs to remain against the cervix for 6 to 8 hours to prevent pregnancy but should not remain for longer than 8 hours to avoid the risk of toxic shock syndrome. The diaphragm should be replaced every 3 months to maintain integrity.

Twenty four hours after a client returns from surgical gastric bypass, the registered nurse (RN) observes large amounts of blood in the nasogastric tube (NGT) cannister. Which assessment finding should the RN report as early signs of hypovolemic shock? Faint pedal pulses. Decrease in blood pressure. Lethargy. Correct Slow breathing.

Decrease in blood pressure. One of the early signs of hypovolemic shock is changes in the client's level of consciousness due to the decrease perfusion to the brain which can manifests as lethargy or confusion.

A client has been told that there is cataract formation over both eyes. Which finding should the nurse expect when assessing the client? Decreased color perception. Presence of floaters. Loss of central vision. Reduced peripheral vision.

Decreased color perception. Decreased color perception occurs with cataract formation. Cataract formation is also associated with blurred vision and a global loss of vision so gradual that the client may not be aware of it.

What is the primary nursing problem for a client with asymptomatic primary syphilis? Acute pain. Risk for injury. Sexual dysfunction. Deficient knowledge.

Deficient knowledge. An asymptomatic client with primary syphilis is most likely unaware of this disease, so to prevent transmission to others and recurrence in the client, the priority nursing diagnosis is deficient knowledge of the disease pathophysiology.

The nurse is caring for a male client who had an inguinal herniorrhaphy 3 hours ago. The nurse determines the client's lower abdomen is distended and assesses dullness to percussion. What is the priority nursing action? Assessment of the client's vital signs. Document the finding as the only action. Determine the time the client last voided. Insert a rectal tube for the passage of flatus.

Determine the time the client last voided. Swelling at the surgical site in the immediate postoperative period can impact the bladder and prostate area causing the client to experience difficulty voiding due to pressure on the urethra. To provide additional data supporting bladder distention, the last time the client voided should be determined next.

The nurse is interviewing a male client with hypertension. Which additional medical diagnosis in the client's history presents the greatest risk for developing a cerebral vascular accident (CVA)? Diabetes mellitus. Hypothyroidism. Parkinson's disease. Recurring pneumonia.

Diabetes mellitus. According to the National Stroke Association (2013), history of diabetes mellitus poses the greatest risk for developing a CVA, 2-4Xs more than those who do not have diabetes mellitus. The reason for this occurrence is related to the excess glucose circulating throughout the body not being utilized by the cells, leading to increased fatty deposits or clots inside the blood vessels in the brain or neck, eventually causing a stroke.

The nurse assesses a client with advanced cirrhosis of the liver for signs of hepatic encephalopathy. Which finding would the nurse consider an indication of progressive hepatic encephalopathy? An increase in abdominal girth. Hypertension and a bounding pulse. Decreased bowel sounds. Difficulty in handwriting.

Difficulty in handwriting A daily record in handwriting may provide evidence of progression of hepatic encephalopathy leading to coma.

A client experiencing uncontrolled atrial fibrillation is admitted to the telemetry unit. Which initial medication should the nurse anticipate administering to the client? Xylocaine. Procainamide. Phenytoin. Digoxin.

Digoxin Digoxin is administered for uncontrolled, symptomatic atrial fibrillation resulting in a decreased cardiac output. Digoxin slows the rate of conduction by prolonging the refractory period of the AV node, thus slowing the ventricular response, decreasing the heart rate, and effecting cardiac output.

When teaching a client with breast cancer about the prescribed radiation therapy for treatment, what information is important to include? Dry, itchy skin changes may occur. There is a possibility of long bone pain. Permanent pigment changes to the breast may result. A low-residue diet may be prescribed to reduce the likelihood of diarrhea.

Dry, itchy skin changes may occur. Side effects from radiation to the breast most often include temporary skin changes such as dryness, tenderness, redness, swelling, and pruritus.

The nurse is caring for a client with a stroke resulting in right-sided paresis and aphasia. The client attempts to use the left hand for feeding and other self-care activities. The spouse becomes frustrated and insists on doing everything for the client. Based on this data, which nursing problem should the nurse document for this client? Situational low self-esteem related to functional impairment and change in role function. Disabled family coping related to dissonant coping style of significant person. Interrupted family processes related to shift in health status of family member. Risk for ineffective therapeutic regimen management related to complexity of care.

Disabled family coping related to dissonant coping style of significant person. A stroke affects the whole family and in this case the spouse probably thinks that she is helping and needs to feel that she is contributing to the client's care. Her help is noted as being incongruent with attempts of self-care by the client thereby disabling family coping.

A 49-year-old female client arrives at the clinic for an annual exam and asks the nurse why she becomes excessively diaphoretic and feels warm during nighttime. What is the nurse's best response? Explain the effect of the follicle-stimulating and luteinizing hormones. Discuss perimenopause and related comfort measures. Assess lung fields and for a cough productive of blood-tinged mucous. Ask if a fever above 101 F (38.3 C) has occurred in the last 24 hours.

Discuss perimenopause and related comfort measures. The perimenopausal period begins about 10 years before menopause with the cessation of menstruation at the average ages of 52 to 54. Lower estrogen levels causes FSH and LH secretion in bursts (surges), which triggers vasomotor instability, night sweats, and hot flashes, so discussions about the perimenopausal body's changes, comfort measures, and treatment options should be provided.

A client who just tested positive for human immunodeficiency virus (HIV) does not appear to hear what the nurse says during post-test counseling. Which information should the nurse offer to facilitate the client's adjustment to HIV infection? Inform the client how to protect sexual and needle-sharing partners. Teach the client about medications that are available for treatment. Identify the need to test others who have had risky contact with the client. Discuss retesting to verify the results, which will ensure continuing contact.

Discuss retesting to verify the results, which will ensure continuing contact. Encouraging retesting supports hope and gives the client time to cope with the diagnosis. Although post-test counseling should include education, retesting encourages the client to maintain medical follow-up and management.

A client with a history of hypertension, myocardial infarction, and heart failure is admitted to the surgical intensive care unit after coronary artery bypass surgery graft (CABG). The nurse determines the client's serum potassium level is 4.5 mEq/L. What action should the nurse implement? Notify the healthcare provider. Decrease the IV solution flow rate. Document the finding as the only action. Administer potassium replacement as prescribed.

Document the finding as the only action. Coronary artery bypass surgery graft (CABG) places a client at risk for hypokalemia from hemodilution, nasogastric suction, or diuretic therapy, so the serum potassium level is maintained between 4 and 5 mEq/L to avoid dysrhythmias. Documentation of the normal finding is indicated at this time.

The unlicensed assistive personnel (UAP) reports that an 87-year-old client who is sitting in a chair at the bedside has an oral temperature of 97.2°F (36.4°C). Which intervention should the nurse implement? Document the temperature reading on the vital sign graphic sheet. Report the temperature to the healthcare provider immediately. Instruct the UAP to take the client's temperature again in 30 minutes. Advise the UAP to assist the client in returning to bed.

Document the temperature reading on the vital sign graphic sheet. A subnormal oral temperature of 97.2°F (36.4°C) is a common finding in elderly clients, so the nurse should document the findings and continue with the plan of care.

The nurse is initiating the client's fourth dose of gentamycin sulfate IV. The health care provider (HCP) has prescribed peak and trough levels. Which is the most important action for the nurse to implement next? Draw the peak 15 minutes before and the trough 15 minutes after the next dose. Draw the peak one hour before and one hour after the next dose. Draw the trough 5 minutes before and the peak 30 minutes after the next dose. Draw the trough 30 minutes before and 30 minutes after the next dose.

Draw the trough 5 minutes before and the peak 30 minutes after the next dose. Peak drug serum levels are achieved 30 minutes after the completion of the IV infusion of gentamicin sulfate. The best time to draw a trough is the closest time to the next prior administration.

Which dietary assessment finding is most important for the nurse to address when caring for a client with diabetic nephropathy? Drinks a six pack of beer every day. Enjoys a hamburger once a month. Eats fortified breakfast cereal daily. Consumes beans and rice every day.

Drinks a six pack of beer every day. Drinking six beers every day is the dietary assessment finding most important for the nurse to address when caring for a client with diabetic nephropathy. The usual can of beer is 12 ounces (355 mL). Clients with diabetes are recommended to drink no more than 12 ounces of beer per day because beer contains carbohydrates that can create unhealthy fluctuations in blood glucose and promote poorglucose control. Nephropathy is exacerbated by poor blood glucose control.

The registered nurse (RN) is caring for an older client who recently experienced a fractured pelvis from a fall. Which assessment finding is most important for the RN to report the healthcare provider? Lower back pain. Headache of 7 on scale 1 to 10. Blood pressure of 140/98. Dyspnea.

Dyspnea A client with a large bone fracture is at risk for intramedullary fat leaking into the blood stream and becoming embolic. Dyspnea is an indication of fat embolism to the lungs and should be reported to the healthcare provider immediately.

A client is admitted to the medical intensive care unit with a diagnosis of myocardial infarction. The client's history indicates the infarction occurred ten hours ago. Which laboratory test result would the nurse expect this client to exhibit? Elevated LDH. Elevated serum amylase. Elevated CK-MB. Elevated hematocrit.

Elevated CK-MB. The cardiac isoenzyme CK-MB (C) is the one of the cardiac markers to indicate myocardial damage in the presence of MI symptoms and after a positive troponin. The troponin levels will elevate within 2-3 hours indicating myocardial ischemia, followed by the CK-MB cardiac markers within 6-9 hours, peaking within 12 to 20 hours after myocardial infarction (MI).

When providing discharge teaching for a client with osteoporosis, the nurse should reinforce which home care activity? A diet low in phosphates. Skin inspection for bruising. Exercise regimen, including swimming. Elimination of hazards to home safety.

Elimination of hazards to home safety. Discussion about fall prevention strategies is imperative for the discharged client with osteoporosis. Advice about safety measures in the home should be provided such as the elimination of throw rugs and proper lighting to minimize trip hazards and falls.

Which intervention should the nurse implement for a female client diagnosed with pelvic relaxation disorder? Describe proper administration of vaginal suppositories and cream. Encourage the client to perform Kegel exercises 10 times daily. Explain the importance of using condoms when having sexual intercourse. Discuss the importance of keeping a diary of daily temperature and menstrual cycle events.

Encourage the client to perform Kegel exercises 10 times daily. Pelvic relaxation disorders are structural disorders resulting from weakening support tissues of the pelvis. Kegel exercises help strengthen the surrounding muscles.

Which postmenopausal client's complaint should the nurse refer to the healthcare provider? Breasts feel lumpy when palpated. History of white nipple discharge. Episodes of vaginal bleeding. Excessive diaphoresis occurs at night.

Episodes of vaginal bleeding. Postmenopausal vaginal bleeding may be an indication of endometrial cancer, which should be reported to the healthcare provider.

A postmenopausal client asks the nurse why she is experiencing discomfort during intercourse. Which response is best for the nurse to provide? Estrogen deficiency causes the vaginal tissues to become dry and thinner. Infrequent intercourse results in the vaginal tissues losing their elasticity. Dehydration from inadequate fluid intake causes vulva tissue dryness. Lack of adequate stimulation is the most common reason for dyspareunia.

Estrogen deficiency causes the vaginal tissues to become dry and thinner. Estrogen deprivation decreases the moisture-secreting capacity of vaginal cells, so vaginal tissues tend to become thinner, drier, and the rugae become smoother which reduces vaginal stretching that contributes to dyspareunia. The discomfort during intercourse, primary cause can be contributed to the decrease in estrogen hormone levels.

A male client receives a local anesthetic during surgery. During the post-operative assessment, the nurse notices the client is slurring his speech. Which action should the nurse take? Determine the client is anxious and allow him to sleep. Evaluate his blood pressure, pulse, and respiratory status. Review the client's pre-operative history for alcohol abuse. Continue to monitor the client for reactivity to anesthesia.

Evaluate his blood pressure, pulse, and respiratory status. Slurred speech in the post-operative client who received a local anesthetic is an atypical finding and may indicate neurological deficits that require further assessment, so obtaining the client's vital signs will provide information about possible cardiovascular complications, such as stroke.

A female client with hyperesthesia on the oncology unit is using a transcutaneous electrical nerve stimulation (TENS) unit for chronic pain. Which nursing activity should the nurse implement instead of delegating to a practical nurse (PN)? Determine the client's level of discomfort using a pain rating scale. Ask the client about her past experience with chronic pain. Observe the client's facial expressions for pain and discomfort. Evaluate the client's ability to adjust the voltage to control pain.

Evaluate the client's ability to adjust the voltage to control pain. The oncology nurse has the knowledge and experience with the use of a transcutaneous electrical nerve stimulation (TENS) unit for chronic pain relief, so the nurse should evaluate the client's skill in effectively controlling the pain by adjusting the voltage.

A client with sickle cell anemia is admitted with severe abdominal pain and the diagnosis is sickle cell crisis. What is the most important nursing action to implement? Limit the client's intake of oral fluids and food. Evaluate the effectiveness of narcotic analgesics. Encourage the client to ambulate as tolerated. Teach the client about prevention of crises.

Evaluate the effectiveness of narcotic analgesics. Pain management is the priority for a client during sickle cell crisis. Continuous narcotic analgesics are the mainstay of pain control, which should be evaluated frequently to determine if the client's pain is adequately controlled.

After a liver biopsy is performed at the bedside, the registered nurse (RN) is assigned the care of the client. Which nursing intervention is most important for the RN to implement? Position client on left side with pillow placed under the costal margin. Assist the client with voiding immediately after the procedure. Evaluate vital signs q10 to 20 minutes for 2 hours after procedure. Ambulate client 3 times in first hour with pillow held at abdomen.

Evaluate vital signs q10 to 20 minutes for 2 hours after procedure. Vital signs should be checked every 10 to 20 minutes to assess for bleeding after biopsy of the liver, which is highly vascular. The client should be positioned on the right side with a pillow or sandbag under the costal margin and supporting the biopsy site. The client should be maintained on bedrest for several hours to decrease the risk of bleeding from the biopsy site.

A client with primary dysmenorrhea has several medications at home. She calls the clinic to ask the registered nurse (RN) which medication should she use for her pain. Which option should the nurse recommend as the first choice in the management of this client's pain? Aspirin. Codeine. Ibuprofen. Acetaminophen.

Ibuprofen Ibuprofen, a nonsteroidal antiinflammatory drug (NSAID) provides the most effective relief for primary dysmenorrhea because it has antiprostaglandin properties.

The nurse is caring for a client with end stage liver disease who is being assessed for the presence of asterixis. Which position should the nurse ask the client to demonstrate to assess for asterixis? Extend the left arm laterally with the left palm upward. Extend the arm, dorsiflex the wrist, and extend the fingers. Extend the arms and hold this position for 30 seconds. Extend arms with both legs adducted to shoulder width.

Extend the arm, dorsiflex the wrist, and extend the fingers. Asterixis (flapping tremor, liver flap) is a hand-flapping tremor that is often seen frequently in hepatic encephalopathy. The tremor is induced by extending the arm and dorsiflexing the wrist, causing rapid, nonrhythmic extension and flexion of the wrist while attempting to hold position.

A client has undergone insertion of a permanent pacemaker. When developing a discharge teaching plan, the nurse writes a goal of, "The client will verbalize symptoms of pacemaker failure." Which symptoms are most important to teach the client? Facial flushing. Fever. Pounding headache. Feelings of dizziness.

Feelings of dizziness. Feelings of dizziness may occur as the result of a decreased heart rate, leading to a decreased cardiac output which may be an indication of pacemaker failure.

A female client with type 2 diabetes mellitus reports dysuria. Which assessment finding is most important for the nurse to report to the healthcare provider? Suprapublic pain and distention. Bounding pulse at 100 beats/minute. Fingerstick glucose of 300 mg/dl. Small vesicular perineal lesions.

Fingerstick glucose of 300 mg/dl. Elevated fingerstick glucose levels needs to be reported tot he healthcare provider, so a plan of care can be adjusted to treat the elevated glucose level. Also elevated glucose levels, spills into the urine and provide a medium for bacterial growth.

While working in the emergency room, the nurse is exposed to a client with active tuberculosis. When should the nurseplan to obtain a tuberculin skin test? Immediately after the exposure. Incorrect Within one week of the exposure. Four to six weeks after the exposure. Three months after the exposure.

Four to six weeks after the exposure. A tuberculin skin test is effective 4 to 6 weeks after an exposure, so the individual with a known exposure should wait 4 to 6 weeks before having a tuberculin skin test.

A client with osteoarthritis receives a prescription for naproxen. Which potential side effect should the nurse discuss with the client about this medication? Sensitivity to sunlight. Muscle fasciculations. Increased urinary frequency. Gastrointestinal disturbance.

Gastrointestinal disturbance. Prostaglandin synthesis inhibitors such as naproxen can have gastrointestinal side effects such as nausea and gastric burning. It is recommended that this medication be taken with food to avoid gastrointestinal upset.

The nurse is caring for a client with a small bowel obstruction. The client is vomiting foul smelling fecal-like material. Which action should the nurse implement? Administer antiemetics every 2 to 3 hours. Position on the left side with knees drawn up. Encourage ice chips sparingly. Give IV fluids with electrolytes.

Give IV fluids with electrolytes. When the bowel is obstructed, electrolytes and fluids are not absorbed, so parenteral fluids with sodium chloride, bicarbonate, and potassium should be administered to prevent electrolyte imbalance and dehydration.

A client who returns to the unit after having a percutaneous transluminal coronary angioplasty (PTCA) complains of acute chest pain. What action should the nurse implement next? Inform the healthcare provider. Obtain a 12-lead electrocardiogram. Give a sublingual nitroglycerin tablet. Administer prescribed analgesic.

Give a sublingual nitroglycerin tablet. After a percutaneous transluminal coronary angioplasty (PTCA), a client who experiences acute chest pain may be experiencing cardiac ischemia related to restenosis, stent thrombosis, or acute coronary syndrome involving any coronary artery. The first action is to administer nitroglycerin to dilate the coronary arteries and increase myocardial oxygenation.

A client with a completed ischemic stroke has a blood pressure of 180/90 mmHg. Which action should the nurse implement? Position the head of the bed (HOB) flat. Withhold intravenous fluids. Administer a bolus of IV fluids. Give an antihypertensive medication.

Give an antihypertensive medication. Most ischemic strokes occur during sleep when baseline blood pressure declines or blood viscosity increases due to minimal fluid intake. Completed strokes usually produce neurologic deficits within an hour, and the client's current elevated blood pressure requires antihypertensive medication.

A client is admitted to the hospital with a medical diagnosis of pneumococcal pneumonia. The nurse knows that the prognosis for gram-negative pneumonias (such as E. coli, Klebsiella, Pseudomonas, and Proteus) is very poor. Which information relates most directly to the prognosis for gram-negative pneumonias? The gram-negative infections occur in the lower lobe alveoli which are more sensitive to infection. Gram-negative organisms are more resistant to antibiotic therapy. Usually occur in healthy young adults who have recently been debilitated by an upper respiratory infection. Gram-negative pneumonias usually affect infants and small children.

Gram-negative organisms are more resistant to antibiotic therapy. Gram-negative organisms are very resistant to drug therapy which makes recovery difficult. Antibiotic resistance has become a world-wide concern and the World Health Organization is keeping a very close surveillance on these occurrences.

The nurse is giving discharge instructions to a client with chronic prostatitis. What instruction should the nurse provide the client to reduce the risk of spreading the infection to other areas of the client's urinary tract? Wear a condom when having sexual intercourse. Avoid consuming alcohol and caffeinated beverages. Empty the bladder completely with each voiding. Have intercourse or masturbate at least twice a week.

Have intercourse or masturbate at least twice a week. The prostate is not easily penetrated by antibiotics and can serve as a reservoir for microorganisms, which can infect other areas of the genitourinary tract. Draining the prostate regularly through intercourse or masturbation decreases the number of microorganisms present and reduces the risk for further infection from stored contaminated seminal fluids.

A middle-aged male client with diabetes continues to eat an abundance of foods that are high in sugar and fat. According to the Health Belief Model, which event is most likely to increase the client's willingness to become compliant with the prescribed diet? He visits his diabetic brother who just had surgery to amputate an infected foot. He is provided with the most current information about the dangers of untreated diabetes. He comments on the community service announcements about preventing complications associated with diabetes. His wife expresses a sincere willingness to prepare meals that are within his prescribed diet.

He visits his diabetic brother who just had surgery to amputate an infected foot. The loss of a limb due to diabetes by a family member should be the strongest event or "cue to action" and is most likely to increase the client's perceived seriousness of the disease.

Which symptoms should the nurse expect a client to exhibit who is diagnosed with a pheochromocytoma? Numbness, tingling, and cramps in the extremities. Headache, diaphoresis, and palpitations. Cyanosis, fever, and classic signs of shock. Nausea, vomiting, and muscular weakness.

Headache, diaphoresis, and palpitations. Pheochromocytoma is a catecholamine secreting non-cancerous tumor of the adrenal medulla, and a headache, profuse sweating and palpitations is the typical triad of symptoms depending upon the relative proportions of epinephrine and norepinephrine secretion. Surgical removal of the tumor is the only treatment.

A client in an ambulatory clinic describes awaking in the middle of the night with difficulty breathing and shortness of breath related to paroxysmal nocturnal dyspnea. Which underlying condition should the registered nurse (RN) identify in the client's history? Chronic bronchitis. Gastroesophageal reflux disease (GERD). Heart failure (HF). Chronic pancreatitis.

Heart failure (HF). Paroxysmal nocturnal dyspnea is classic sign of heart failure and is secondary to fluid overload associated with heart failure which causes pulmonary edema.

A young adult female reports that she is experiencing a lack of appetite, hypersomnia, stress incontinence, and heart palpitations. Which symptom is characteristic of premenstrual syndrome? Heart palpitations. Anorexia. Hypersomnia. Stress incontinence.

Heart palpitations. Characteristic features of premenstrual syndrome include heart palpitations, sleeplessness, increased appetite and food cravings, and oliguria or enuresis.

An 81-year-old male client has emphysema. He lives at home with his cat and manages self-care with no difficulty. When making a home visit, the nurse notices that his tongue is somewhat cracked and his eyeballs are sunken into his head. Which nursing intervention is indicated? Help the client to determine ways to increase his fluid intake. Obtain an appointment for the client to see an ear, nose, and throat specialist. Schedule an appointment with an allergist to determine if the client is allergic to the cat. Encourage the client to slightly increase his use of oxygen at night and to always use humidified oxygen.

Help the client to determine ways to increase his fluid intake. The nurse should suggest creative methods to increase the intake of fluids, such as having disposable fruit juices readily available. Clients with COPD should be encouraged to have at least three liters of fluids a day to help keep their mucus thin. As the disease progresses, these clients often reduce fluid intake because of shortness of breath experienced while drinking and due to the fact,that they may be on diuretics related to heart involvement with the disease and may purposely limit their fluid intake to decrease the need for elimination.

The nurse is receiving report from surgery about a client with a penrose drain who is to be admitted to the postoperative unit. Before choosing a room for this client, which information is most important for the nurse to obtain? If suctioning will be needed for drainage of the wound. If the family would prefer a private or semi-private room. Prescription for removal of the drain. If the client's wound is infected.

If the client's wound is infected. Penrose drains provide a sinus tract or opening and are often used to provide drainage of an abscess. The fact that the client has a penrose drain should alert the nurse to the possibility that the client is infected. To avoid contamination of another postoperative client, it is most for the nurse to verify the condition of the wound and if infected, important to place client in a private room.

Which physical assessment finding should the nurse anticipate in a client with long-term gastroesophagealreflux disease (GERD)? Hoarseness. Dry mouth. Mouth ulcers. Weight loss.

Hoarseness Dyspepsia and regurgitation are the main symptoms of gastroesophageal reflux disease (GERD); however, hoarseness is one of the most common long-term symptoms of GERD due to the irritation of the reflux of gastric secretions.

Which nail color alteration should the nurse expect to observe in a client with chronic kidney disease? Horizontal white banding. Diffuse blue discoloration. Diffuse brown discoloration. Thin, dark red vertical lines.

Horizontal white banding. Fingernails and toenails can be affected by chronic kidney disease. This condition may cause horizontal white lines or bands (leukonychia) to appear on the nails.

Which sexually transmitted infection (STI) should the nurse include in a client's teaching plan that increases the risk for cervical cancer? Neisseria gonorrhoea. Chlamydia trachomatis. Herpes simplex virus. Human papillomavirus.

Human papillomavirus. According to the CDC (2017), it is estimated at least 80% of all women who are sexually active will contract the Human papillomavirus (HPV) in their lifetime. Certain types of HPV have been suspected to cause cervical cancer and HPV strain 16 and 18 have been identified to cause 70% of cervical cancers.

Which statement made by a client with chronic pancreatitis indicates that further education is needed? I will cut back on smoking cigarettes daily. I will avoid drinking caffeinated beverages. I will rest frequently and avoid vigorous exercise. I will eat a bland, low-fat, high-protein diet.

I will cut back on smoking cigarettes daily. To prevent exacerbations of chronic pancreatitis, clients should be instructed to avoid nicotine entirely. Additional teaching includes avoiding caffeinated beverages, resting frequently as needed, and eating a bland diet low fat and high in protein.

An older female client is admitted with atrophic vaginitis and perineal cutaneous candidiasis. Which is the priority nursing diagnosis for this client? Risk for injury. Impaired comfort. Disturbed body image. Ineffective health maintenance.

Impaired comfort. In menopausal women, the vaginal mucous membrane responds to low estrogen levels causing the vaginal walls to become thinner, drier, and susceptible to infection, which leads to atrophic vaginitis. Perineal cutaneous candidiasis contributes to other manifestations of vaginal infections, such as vaginal irritation, burning, pruritus, increased leukorrhea, bleeding, and dyspareunia, which supports the primary nursing diagnosis, "impaired comfort."

A client is placed on a mechanical ventilator following a cerebral hemorrhage, and vecuronium bromide 0.04 mg/kg q12 hours IV is prescribed. Which is the priority nursing diagnosis for this client? Impaired communication related to paralysis of skeletal muscles. High risk for infection related to increased intracranial pressure. Potential for injury related to impaired lung expansion. Social isolation related to inability to communicate.

Impaired communication related to paralysis of skeletal muscles. To increase the client's tolerance of endotracheal intubation and/or mechanical ventilation, a skeletal-muscle relaxant, such as vecuronium, is usually prescribed. Impaired communication is a serious outcome because the client cannot communicate his/her needs due to intubation and diaphragmatic paralysis caused by the drug.

A client is placed on a mechanical ventilator following a cerebral hemorrhage, and vecuronium bromide 0.04 mg/kg q12 hours IV is prescribed. Which is the priority nursing diagnosis for this client? Impaired communication related to paralysis of skeletal muscles. High risk for infection related to increased intracranial pressure. Potential for injury related to impaired lung expansion. Social isolation related to inability to communicate.

Impaired communication related to paralysis of skeletal muscles. To increase the client's tolerance of endotracheal intubation and/or mechanical ventilation, a skeletal-muscle relaxant, such as vecuronium, is usually prescribed. Impaired communication is a serious outcome because the client cannot communicate his/her needs due to intubation and diaphragmatic paralysis caused by the drug.

A 32-year-old female client complains of severe abdominal pain each month before her menstrual period, painful intercourse, and painful defecation. Which additional history should the nurse obtain that is consistent with the client's complaints? Frequent urinary tract infections. Inability to get pregnant. Premenstrual syndrome. Chronic use of laxatives.

Inability to get pregnant. Dysmenorrhea, dyspareunia, and difficulty or painful defecation are common symptoms of endometriosis, which is the abnormal displacement of endometrial tissue in the dependent areas of the pelvic peritoneum. A history of infertility is another common finding associated with endometriosis.

The nurse is providing dietary instructions to a 68-year-old client who is at high risk for development of coronary heart disease (CHD). Which information should the nurse include? Limit dietary selection of cholesterol to 300 mg per day. Increase intake of soluble fiber to 10 to 25 grams per day. Decrease plant stanols and sterols to less than 2 grams/day. Ensure saturated fat is less than 30% of total caloric intake.

Increase intake of soluble fiber to 10 to 25 grams per day. To reduce risk factors associated with coronary heart disease, the daily intake of soluble fiber should be increased to between 10 and 25 grams per day. According to the American Heart Association, soluble fibers helps reduce LDL cholesterol levels.

A client with rheumatoid arthritis is prescribed piroxicam, a nonsteroidal anti-inflammatory drug (NSAID). Which effect is characteristic of NSAIDs used for treating rheumatoid arthritis? Production of replacement cartilage is stimulated. Further destruction of the articular cartilage is prevented. Inflammation is reduced by inhibiting prostaglandin synthesis. Bradykinin is inhibited, thereby reducing acute and chronic pain.

Inflammation is reduced by inhibiting prostaglandin synthesis. Nonsteroidal anti-inflammatory drugs (NSAIDs), used for treating rheumatoid arthritis, work by inhibiting the synthesis of prostaglandins and providing relief from the associated pain.

The nurse is working with a 71-year-old obese client with bilateral osteoarthritis (OA) of the hips. Which recommendation should the nurse make that is most beneficial in protecting the client's joints? Increase the amount of calcium intake in the diet. Apply alternating heat and cold therapies. Initiate a weight-reduction diet to achieve a healthy body weight. Use a walker for ambulation to lessen weight-bearing on the hips.

Initiate a weight-reduction diet to achieve a healthy body weight. Achieving a healthy weight is critical to protect the joints of clients with osteoarthritis (OA). Weight loss for obese clients will take off the excess pressure that weight bearing joints such as the hips and knees are exposed to and reduce the wear and tear of the joints.

A client who has heart failure is admitted with a serum potassium level of 2.9 mEq/L (2.9 mmol/L). Which action is most important for the nurse to implement? Give 20 mEq of potassium chloride. Initiate continuous cardiac monitoring. Arrange a consultation with the dietician. Teach about the side effects of diuretics.

Initiate continuous cardiac monitoring. Hypokalemia (normal 3.5 to 5 mEq/L [3.5 to 5 mmol/L]) causes changes in myocardial irritability and ECG waveform, so it is most important for the nurse to initiate continuous cardiac monitoring to identify ventricular ectopy or other life-threatening dysrhythmias. After cardiac monitoring is initiated, then the potassium chloride should be given so that the effects of potassium replacement on the cardiac rhythm can be monitored.

A client has a staging procedure for cancer of the breast and ask the nurse which type of breast cancer has the poorest prognosis. Which information should the nurse offer the client? Stage II. Invasive infiltrating ductal carcinoma. T1N0M0. Inflammatory with peau d'orange.

Invasive infiltrating ductal carcinoma. Inflammatory breast cancer onset is very rapid and a very rare form of breast cancer and is considered the most aggressive form of breast malignancies. It is often mistaken for a breast infection because it has a thickened appearance like an orange peel (peau d'orange), causing the breast to become swollen and tender.

The nurse is caring for a client after a transurethral resection of the prostate (TURP) and determines the client's urinary catheter is not draining. What should the nurse implement? Reposition the catheter drainage tubing. Encourage the client to drink oral fluids. Irrigate the catheter. Change drainage unit tubing.

Irrigate the catheter. Obstruction urinary flow after a TURP is most often due to blood clots, and sterile irrigation should be implemented to remove the clots that are blocking the catheter.

The healthcare provider prescribes high-protein, high-fat, low-carbohydrate diet with limited fluids during meals for a client recovering from gastric surgery. The client asks the nurse what the purpose is for this type of diet. Which rationale should be included in the nurse's explanation to this client? It is quickly digested. It does not cause diarrhea. It does not dilate the stomach. It is slow to leave the stomach.

It is slow to leave the stomach. This type of diet is slowly digested and is slow to leave the stomach, thereby the possibility of dumping syndrome is reduced as a result of its density from proteins and fats, and the reduction of fluids.

A client's susceptibility to ulcerative colitis is most likely due to which aspect in the client's history? Jewish European ancestry. H. pylori bowel infection. Family history of irritable bowel syndrome. Incorrect Age between 25 and 55 years.

Jewish European ancestry. Ulcerative colitis is 4 to 5 times more common among individuals of Jewish European or Ashkenazi ancestry.

Which description of pain is consistent with a diagnosis of rheumatoid arthritis? Joint pain is worse in the morning and involves symmetric joints. Joint pain is better in the morning and worsens throughout the day. Joint pain is consistent throughout the day and is relieved by pain medication. Joint pain is worse during the day and involves unilateral joints.

Joint pain is worse in the morning and involves symmetric joints Rheumatoid arthritis (RA) is an autoimmune disease that causes joint pain and swelling. RA is characterized by pain that is worse when arising and involves symmetric joints.

A client receiving cholestyramine for hyperlipidemia should be evaluated for which vitamin deficiency? K. B12. B6. C.

K Cholestyramine is administered to help lower the triglycerides levels. Side effects clients should be monitored for include increased prothrombin time and prolonged bleeding times which would alert the nurse to a vitamin K deficiency. These drugs reduce absorption of the fat soluble (lipid) vitamins A, D, E, and K.

A nurse is preparing to insert an IV catheter after applying an eutectic mixture of lidocaine and prilocaine (EMLA), a topical anesthetic cream. What action should the nurse take to maximize its therapeutic effect? Rub a liberal amount of cream into the skin thoroughly. Cover the skin with a gauze dressing after applying the cream. Leave the cream on the skin for 1 to 2 hours before the procedure. Use the smallest amount of cream necessary to numb the skin surface.

Leave the cream on the skin for 1 to 2 hours before the procedure. Topical anesthetic creams, such as EMLA, should be applied to the puncture site at least 60 minutes to 2 hours before the insertion of an IV catheter.

In planning care for a client with an acute stroke resulting in right-sided hemiplegia, which positioning should the nurse should use to maintain optimal functioning? Mid-Fowler's with knees supported. Supine with trochanter rolls to the hips. Sim's position alternated with right lateral position q2 hours. Left lateral, supine, brief periods on the right side, and prone.

Left lateral, supine, brief periods on the right side, and prone. After an acute stroke, a positioning and turning schedule that minimizes lying on the affected side is recommended because it prevents impaired circulation and reduces pain. The prone position helps prevent flexion contractures of the hips and prepares the client for optimal functioning and ambulating.

A client with osteoarthritis requests information from the nurse about what type of exercise regimen would be most beneficial for him. The nurse should communicate which information? Low impact exercise like walking, swimming, and water aerobics. Repetitive strength-building exercises with weights or resistance bands. Circuit training alternating with frequent rest periods. High-impact aerobic exercise.

Low impact exercise like walking, swimming, and water aerobics. Low impact exercises that do not put additional pressure and strain or cause further harm to damaged joints, such as walking or swimming, are most beneficial to clients with osteoarthritis.Strength-building exercises, circuit training, and high-impact aerobics may cause too much stress on the joint areas and subsequently increase inflammation and damage.

A client's prostate-specific antigen (PSA) exam result showed a PSA density of 0.13 ng/ml. Which conclusion regarding this lab data is accurate? Probable prostatitis. Low risk for prostate cancer. The presence of cancer cells. Biopsy of the prostate is indicated.

Low risk for prostate cancer. Clients with a PSA density less than 0.15 ng/ml are considered at low risk for prostate cancer.

A client is admitted for further testing to confirm sarcoidosis. Which diagnostic test provides definitive information that the nurse should report to the healthcare provider? Lung tissue biopsy. Positive blood cultures. Magnetic resonance imaging (MRI). Computerized tomography (CT) of the thorax.

Lung tissue biopsy. Sarcoidosis is an inflammatory condition that is characterized by the formation of widespread granulomatous lesions involving a pulmonary primary site. Although chest radiography identifies sarcoidosis, lung tissue biopsy obtained by bronchoscopy or bronchoalveolar lavage provides definitive confirmation.

The nurse is planning care to prevent complications for a client with multiple myeloma. Which intervention is most important for the nurse to include? Safety precautions during activity. Assess for changes in size of lymph nodes. Maintain a fluid intake of 3 to 4 L per day. Administer narcotic analgesic around the clock.

Maintain a fluid intake of 3 to 4 L per day. Multiple myeloma is a malignancy of plasma cells that infiltrate bone causing demineralization and hypercalcemia, so maintaining a urinary output of 1.5 to 2 L per day requires an intake of 3 to 4 L to promote excretion of serum calcium.

The registered nurse (RN) is caring for a client with aplastic anemia who is hospitalized for weight loss and generalized weakness. Laboratory values show a white blood count (WBC) of 2,500/mm 3 and a platelet countof 160,000/mm 3. Which intervention is the primary focus in the client's plan of care for the RN to implement? Assist with frequent ambulation. Encourage visitors to visit. Maintain strict protective precautions. Avoid peripheral injections.

Maintain strict protective precautions. The client should be under strict protective transmission precautions because the WBC values are low and normal WBC levels are 4,000-10,000/mm3, so the client is an increased high risk for infection.

The nurse is providing instructions about log rolling to a client who returns to the postoperative unit after a lumbar laminectomy. Which explanation should the nurse give the client about this technique? Helps to minimize pain and anxiety. Maintains correct spinal alignment to protect the surgical area. Prevents dizziness while stabilizing the spine. Allows the nurse to move the client freely without assistance.

Maintains correct spinal alignment to protect the surgical area. Log rolling technique maintains the spine in a straight superior-inferior plane and aligns the spine without movement while protecting the surgical area. This is particularly important when the procedure involves bone grafts that may take several weeks for the bone to fuse.

Despite several eye surgeries, a 78-year-old client who lives alone has persistent vision problems. The visiting nurse is discussing home safety hazards with the client. The nurse suggests that the edges of the steps be painted which color? Black. White. Light green. Medium yellow.

Medium yellow The color yellow is the easiest for a person with failing vision to see.

Which preexisting diagnosis places a client at greatest risk of developing superior vena cava syndrome? Carotid stenosis. Steatosis hepatitis. Metastatic cancer. Clavicular fracture.

Metastatic cancer. Superior vena cava syndrome occurs when the superior vena cava (SVC) is compressed by outside structures, such as a growing tumor that impedes the return blood flow to the heart. Superior vena cava syndrome is likely to occur with metastatic cancer from a primary tumor in the upper lobe of the right lung that compresses the superior vena cava.

The nurse is caring for a client scheduled to undergo insertion of a percutaneous endoscopic gastrostomy (PEG) tube. The client asks the nurse to explain how a PEG tube differs from a gastrostomy tube (GT). Which explanation best describes how they are different? Method of insertion. Location of the tubes. Diameter of the tubes. Procedure for feedings.

Method of insertion. The best explanation of how a percutaneous endoscopic gastrostomy (PEG) tube differs from a gastrostomy tube (GT) is by the method of insertion. GT insertion involves making an incision in the wall of the abdomen and suturing the tube to the gastric wall. A PEG tube is more commonly used as it does not require general anesthesia and is less invasive. Insertion is performed with endoscopic visualization through the esophagus into the stomach and then pulled through a small incision in the abdominal wall. It is held in place by a tiny plastic device called a "bumper" that holds the g-tube in place with a small water-filled balloon securing it against the abdominal wall.

During assessment of a client with amyotrophic lateral sclerosis (ALS), which finding should the nurse identify when planning care for this client? Muscle weakness. Urinary frequency. Abnormal involuntary movements. A decline in cognitive function.

Muscle weakness. Amyotrophic lateral sclerosis (ALS) is characterized by a degeneration of motor neurons in the brainstem and spinal cord and is manifested by muscle weakness and wasting.

How should the nurse position the electrodes for modified chest lead one (MCL I) telemetry monitoring? Positive polarity right shoulder, negative polarity left shoulder, ground left chest nipple line. Positive polarity left shoulder, negative polarity right chest nipple line, ground left chest nipple line. Positive polarity right chest nipple line, negative polarity left chest nipple line, ground left shoulder. Negative polarity left shoulder, positive polarity right chest nipple line, ground left chest nipple line.

Negative polarity left shoulder, positive polarity right chest nipple line, ground left chest nipple line. In MCL I monitoring, the positive electrode is placed on the client's mid-chest to the right of the sternum, and the negative electrode is placed on the upper left part of the chest. The ground may be placed anywhere, but is usually placed on the lower left portion of the chest.

Which assessment finding should most concern the nurse who is monitoring a client two hours after a thoracentesis? New onset of coughing. Low resting heart rate. Distended neck veins. Decreased shallow respirations.

New onset of coughing. A pneumothorax (partial or complete lung collapse) is the potential complication of a thoracentesis. Manifestations of a pneumothorax include new onset of a nagging cough, tachycardia, and an increased shallow respiration rate.

Which assessment finding by the nurse during a client's clinical breast examination requires follow-up? Newly retracted nipple. A thickened area where the skin folds under the breast. Whitish nipple discharge. Tender lumpiness noted bilaterally throughout the breasts.

Newly retracted nipple. A newly retracted nipple, compared to a life-long finding, may be an indication of breast cancer and requires additional follow-up.

A client with a 16-year history of diabetes mellitus is having renal function tests because of recent fatigue, weakness, elevated blood urea nitrogen, and serum creatinine levels. Which finding should the nurse conclude as an early symptom of renal insufficiency? Dyspnea Nocturia Confusion. Stomatitis

Nocturia As the glomerular filtration rate decreases in early renal insufficiency, metabolic waste products, including urea, creatinine, and other substances, such as phenols, hormones, and electrolytes, accumulate in the blood. In the early stage of renal insufficiency, polyuria results from the inability of the kidneys to concentrate urine and contributes to nocturia.

The nurse is assessing a client who smokes cigarettes and has been diagnosed with emphysema. Which finding would the nurse expect this client to exhibit? A decreased total lung capacity. Normal arterial blood gases. Normal skin coloring. An absence of sputum.

Normal skin coloring. The differentiation between the "pink puffer" and the "blue bloater" is a well-known method of differentiating clients exhibiting symptoms of emphysema (normal color but puffing respirations) from those exhibiting symptoms of chronic bronchitis (edematous, cyanotic, shallow respirations).

The nurse is preparing an adult client for an upper gastrointestinal (UGI) series. Which information should the nurse include in the teaching plan? The xray procedure may last for several hours. A nasogastric tube (NGT) is inserted to instill the barium. Enemas are given to empty the bowel after the procedure. Nothing by mouth is allowed for 6 to 8 hours before the study.

Nothing by mouth is allowed for 6 to 8 hours before the study. The client should be NPO, to include smoking or chewing gum for at least 6 hours before the UGI study.

A male client with sickle cell anemia, who has been hospitalized for another health problem, tells the nurse he has had an erection for over 4 hours. What action should the nurse implement first? Notify the client's healthcare provider. Document the finding in the client record. Prepare a warm enema solution for rectal instillation. Obtain a large bore needle for aspiration of the corpora cavernosa.

Notify the client's healthcare provider. Priapism, a urologic emergency, is common during sickle cell crisis due to sickle cells clogging the microcirculation in the penis, causing a reduction of blood flow and oxygenation to the penis, so the healthcare provider should be first notified immediately. The prescribed therapy may consist of noninvasive measures such as applying ice to the penis, instilling a warm solution enema to increase outflow in the corpora cavernosa and giving pain medications. If noninvasive measures do not work, then needle aspiration of the corpora cavernosa is implemented by the healthcare provider.

A client who is admitted to the coronary care unit with a myocardial infarction (MI) begins to develop increased pulmonary congestion, an increase in heart rate from 80 to 102 beats per minute, and cold, clammy skin. What action should the nurse implement? Notify the healthcare provider. Increase the IV flow rate. Place the client in the supine position. Prepare the client for an emergency echocardiography.

Notify the healthcare provider. Increased pulmonary congestion, increased heart rate, and cold, clammy skin in a client with a myocardial infarction indicate impending cardiogenic shock related to heart failure, a common complication of MI. The healthcare provider should be notified immediately for emergency interventions of this life-threatening complication.

During the assessment of a client who is 24 hours post-hemicolectomy with a temporary colostomy, the nurse determines that the client's stoma is dry and dark red in color. What action should the nurse implement? Notify the surgeon. Document the assesment. Secure a colostomy pouch over the stoma. Place petrolatum gauze dressing over the stoma.

Notify the surgeon. The stoma should appear reddish pink and moist, which indicates circulatory perfusion to the surgical diversion of the intestine. If the stoma becomes dry, firm, flaccid, or is dark red or purple, the stoma is ischemic, and the surgeon should be notified immediately.

Which client should the nurse recognize as most likely to experience sleep apnea? Middle-aged female who takes a diuretic nightly. Obese older male client with a short, thick neck. Adolescent female with a history of tonsillectomy. School-aged male with a history of hyperactivity disorder.

Obese older male client with a short, thick neck. Sleep apnea is characterized by lack of respirations for 10 seconds or more during sleep and is due to the loss of pharyngeal tone which allows the pharynx to collapse during inspiration and obstructs air flow through the nose and mouth. Risk factors which increase the condition of sleep apnea include: excessive weight, increases the risk 4 times more than normal weighing individuals; neck circumference, thicker necks have narrower airways; individuals with inherited narrower airways; males in general are more prone to sleep apnea; females risk increase with being overweight and post-menopausal; increased age (geriatrics); family history; use of alcohol, sedatives or tranquilizers; smokers and those who suffer from nasal allergies.

A 67-year-old woman who lives alone tripped on a rug in her home and fractured her hip. Which predisposing factor probably led to the fracture in the proximal end of her femur? Failing eyesight resulting in an unsafe environment. Renal osteodystrophy resulting from chronic renal failure. Osteoporosis resulting from hormonal changes. Cardiovascular changes resulting in small strokes which impair mental acuity.

Osteoporosis resulting from hormonal changes. The most common cause of a fractured hip in elderly women is osteoporosis, resulting from reduced calcium in the bones as a result of hormonal changes in later life.

The nurse is assessing a client with a cuffed tracheostomy tube in place who is breathing spontaneously. To evaluate if the client can tolerate cuff deflation to promote speaking and swallowing, which action should the nurse implement? Ask the client to try to speak. Assess for respiratory distress. Auscultate for pulmonary crackles after the client drinks a small amount of clear water. Observe the client for coughing colored sputum after drinking a small amount of colored water.

Observe the client for coughing colored sputum after drinking a small amount of colored water. To evaluate the risk for aspiration after the cuff is deflated, the client should be instructed to swallow a small amount of colored water, then the client should be observed for coughing up colored sputum, or the tracheostomy should be suctioned for the presence of colored water.

Which condition should the nurse suspect when a client reports vaginal dryness during intercourse? Obstructed Bartholin's glands. Hyperactive sebaceous glands. Infected bulbourethral glands. Strangulated prostate gland.

Obstructed Bartholin's glands. Bartholin's glands are located posteriorly on each side of the vaginal opening; they secrete lubrication fluid during sexual excitement. The nurse should suspect obstructed Bartholin's glands when a client reports vaginal dryness during intercourse.

Which action should the nurse implement on the scheduled day of surgery for a client with type 1 diabetes mellitus (DM)? Obtain a prescription for an adjusted dose of insulin. Administer an oral anti-diabetic agent. Give an insulin dose using parameters of a sliding scale. Withhold insulin while the client is NPO.

Obtain a prescription for an adjusted dose of insulin. Stressors, such as surgery, increase serum glucose levels. A client with type 1 DM who is NPO for scheduled surgery should receive a prescribed adjusted dose of insulin.

A 40-year-old female client has a history of smoking. Which finding should the nurse identify as a risk factor for myocardia infarction? Oral contraceptives. Senile osteopenia. Levothyroxine therapy. Pernicious anemia.

Oral contraceptives. Women older than 35 years old who smoke and take oral contraceptives have an increased risk of myocardial infarction or stroke.

A client with chest pain, dizziness, and vomiting for the last 2 hours is admitted for evaluation for Acute Coronary Syndrome (ACS). Which cardiac biomarker should the registered nurse (RN) anticipate to be elevated if the client experienced myocardial damage? Creatine Kinase (CK-MB). Serum troponin. Myoglobin. Ischemia modified albumin.

Serum troponin. Troponin is the most sensitive and specific test for myocardial damage. Troponin elevation is more specific than CK-MB.

An older adult male client comes to the geriatric screening clinic complaining of pain in his left calf. The nurse notices a reddened area on the calf of his right leg which is warm to the touch and suspects it might be thrombophlebitis. Which type of pain would further confirm this suspicion? Pain in the calf awakening him from a sound sleep. Calf pain on exertion which stops when standing in one place. Pain in the calf upon exertion which is relieved by rest and elevating the extremity. Pain upon arising in the morning which is relieved after some stretching and exercise.

Pain in the calf upon exertion which is relieved by rest and elevating the extremity. Thrombophlebitis pain is relieved by rest and elevation of the extremity. It typically occurs with exercise at the site of the thrombus, and is aggravated by placing the extremity in a dependent position, such as standing in one place.

A client with acute osteomyelitis has undergone surgical debridement of the diseased bone and asks the nurse how long will antibiotics have to be administered. Which information should the nurse communicate? Oral antibiotics for 2 to 4 months, then for dental procedure prophylaxis. Parenteral antibiotics for 4 to 6 weeks, then oral antibiotics for up to 1 year. Parenteral antibiotics for 4 to 8 weeks, then oral antibiotics for 4 to 8 weeks. Parenteral antibiotics for 2 to 3 weeks, then oral antibiotics for 4 weeks.

Parenteral antibiotics for 4 to 8 weeks, then oral antibiotics for 4 to 8 weeks. Treatment of acute osteomyelitis requires administration of high doses of parenteral antibiotics for 4 to 8 weeks, followed by oral antibiotics for another 4 to 8 weeks.

The nurse obtains a client's history that includes right mastectomy and radiation therapy for breast cancer 10 years ago. Which current health problem should the nurse consider is a consequence of the radiation therapy? Asthma. Myocardial infarction. Chronic esophagitis with gastroesophageal reflux. Pathologic fracture of two ribs on the right chest.

Pathologic fracture of two ribs on the right chest. The ribs lie in the radiation pathway and lose density over time, becoming thin and brittle, so the occurrence of two right-sided ribs with pathological fractures resulting without evidence of trauma is related to radiation damage.

What instruction should the nurse give a client who is diagnosed with fibrocystic changes of the breast? Observe cyst size fluctuations as a sign of malignancy. Use estrogen supplements to reduce breast discomfort. Notify the healthcare provider if whitish nipple discharge occurs. Perform a breast self-exam (BSE) procedure monthly.

Perform a breast self-exam (BSE) procedure monthly. Fibrocystic changes in the breast are related to excess fibrous tissue, proliferation of mammary ducts and cyst formation that cause edema and nerve irritation. These changes obscure typical diagnostic tests, such as mammography, due to an increased breast density. Women with fibrocystic breasts should be instructed to carefully perform monthly BSE and consider changes in any previous "lumpiness." Fibrocystic disease does not increase the risk of breast cancer. Cyst size fluctuates with the menstrual cycle, and typically lessens after menopause, and responds with a heightened sensitivity to circulating estrogen.

Which intervention should the nurse plan to implement when caring for a client who has just undergone a right above-the-knee amputation? Maintain the residual limb on three pillows at all times. Place a large tourniquet at the client's bedside. Apply constant, direct pressure to the residual limb. Do not allow the client to lie in the prone position.

Place a large tourniquet at the client's bedside. A large tourniquet should be placed in plain sight at the client's bedside, in the event severe bleeding occurs. The purpose is to have the tourniquet available to apply to the residual limb to control bleeding if hemorrhaging was to occur.

When teaching diaphragmatic breathing to a client with chronic obstructive pulmonary disease (COPD), which information should the nurse provide? Place a small book or magazine on the abdomen and make it rise while inhaling deeply. Purse the lips while inhaling as deeply as possible and then exhale through the nose. Wrap a towel around the abdomen and push against the towel while forcefully exhaling. Place one hand on the chest, one hand the abdomen and make both hands move outward.

Place a small book or magazine on the abdomen and make it rise while inhaling deeply. Diaphragmatic or abdominal breathing uses the diaphragm instead of accessory muscles to achieve maximum inhalation and to slow the respiratory rate. The client should protrude the abdomen on inhalation and contract it with exhalation, so placing a book or magazine, helps the client visualize the rise and fall of the abdomen.

The registered nurse (RN) is caring for a client who has a closed head injury from a motor vehicle collision. Which finding would indicate to the nurse that the client is at risk for diabetes insipidus (DI)? High fever. Low blood pressure. Muscle rigidity. Polydipsia

Polydipsia A characteristic finding of DI is excretion of large quantities of urine (5 to 20L/day), and most clients compensate for fluid loss by drinking large amounts of water (polydipsia). DI can occur when there has been damage or injury to the pituitary gland or hypothalamus as a result of head trauma, tumor or an illness such as meningitis. This damage interrupts the ADH production, storage and release causing the excessive urination and thirst.

In assessing a client diagnosed with primary hyperaldosteronism, the nurse expects the laboratory test results to indicate a decreased serum level of which substance? Sodium. Antidiuretic hormone. Potassium. Glucose.

Potassium Clients with primary hyperaldosteronism exhibit a profound decline in the serum levels of potassium (hypokalemia). Hypertension, along with the hypokalemia are the most prominent and universal signs for this condition. If both of these findings are present, there is a 50% likelihood the client will be diagnosed with hyperaldosteronism.

The nurse is assessing a client with chronic kidney disease (CKD). Which finding is most important for the nurse to respond to first? Potassium 6.0 mEq. Daily urine output of 400 ml. Peripheral neuropathy. Uremic fetor.

Potassium 6.0 mEq. When assessing a client with chronic kidney disease (CKD), hyperkalemia (normal serum level, 3.5 to 5.5 mEq) is a serious electrolyte disorder that can cause fatal arrhythmias, so the elevation of the potassium level is a nursing priority.

The registered nurse (RN) is assisting the healthcare provider (HCP) with the removal of a chest tube. Which intervention has the highest priority and should be anticipated by the RN after the removal of the chest tube? Prepare the client for chest x-ray at the bedside. Review arterial blood gases after removal. Elevate the head of bed to 45 degrees. Incorrect Assist with disassembling the drainage system.

Prepare the client for chest x-ray at the bedside. A chest x-ray should be performed immediately after the removal of a chest tube to ensure lung expansion has been maintained after its removal.

Small bowel obstruction is a condition characterized by which finding? Severe fluid and electrolyte imbalances. Correct Metabolic acidosis. Ribbon-like stools. Incorrect Intermittent lower abdominal cramping.

Severe fluid and electrolyte imbalances. Among the findings characteristic of a small bowel obstruction is the presence of severe fluid and electrolyte imbalances.

The nurse should explain to a client with lung cancer that pleurodesis is performed to achieve which expected outcome? Prevent the formation of effusion fluid. Remove fluid from the intrapleural space. Debulk tumor to maintain patency of air passages. Relieve empyema after pneumonectomy.

Prevent the formation of effusion fluid. Instillation of a sclerosing agent to create pleurodesis is aimed at preventing the formation of a pleural effusion by causing the pleural spaces sealed together, thereby preventing the accumulation of pleural fluid.

A splint is prescribed for nighttime use by a client with rheumatoid arthritis. Which statement by the nurse provides the most accurate explanation for use of the splints? Prevention of deformities. Avoidance of joint trauma. Relief of joint inflammation. Improvement in joint strength.

Prevention of deformities. Splints may be used at night by clients with rheumatoid arthritis to prevent deformities caused by muscle spasms and contractures.

During a health fair, a 72-year-old male client tells the nurse that he is experiencing shortness of breath. Auscultation reveals crackles and wheezing in both lungs. Suspecting that the client might have chronic bronchitis, which classic symptom would the nurse expect this client to have? Racing pulse with exertion. Clubbing of the fingers. An increased chest diameter. Productive cough with grayish-white sputum.

Productive cough with grayish-white sputum. Chronic bronchitis, one of the diseases comprising the diagnosis of chronic obstructive pulmonary disease (COPD), is characterized by a productive cough with grayish-white sputum.

The registered nurse (RN) is teaching a client who is newly diagnosed with emphysema how to perform pursed lip breathing. What is the primary reason for teaching the client this method of breathing? Decreases respiratory rate. Increases O 2 saturation throughout the body. Conserves energy while ambulating. Promotes CO 2 elimination.

Promotes CO 2 elimination. Pursed lip breathing helps eliminate CO2 by increasing positive pressure within the alveoli increasing the surface area of the alveoli making it easier for the O2 and CO2 gas exchange to occur .

During the initial outbreak of genital herpes simplex in a female client, what should be the nurse's primary focus in planning care? Promotion of comfort. Prevention of pregnancy. Instruction in condom use. Information about transmission.

Promotion of comfort. The initial outbreak of genital herpes simplex in a woman causes severe discomfort. Promotion of comfort is the first priority.

A client with heart disease is on a continuous telemetry monitor and has developed sinus bradycardia. In determining the possible cause of the bradycardia, the nurse assesses the client's medication record. Which medication is most likely the cause of the bradycardia? Propanolol. Captopril. Furosemide. Dobutamine.

Propanolol Propanolol is a beta adrenergic blocking agent, which causes decreased heart rate and decreased contractility.

A young adult male is diagnosed with Stage 4 Hodgkin's lymphoma in the abdominopelvic region and is scheduled for radiation therapy (RT). The client expresses concern about becoming infertile. How should the nurse respond? Propose sperm banking before RT then artificial insemination is an option. Suggest adoption when the client is in remission or ready for parenting. Tell the client that infertility is a non-reversible side effect of radiotherapy. Explain that sperm production will be suppressed after radiotherapy is over.

Propose sperm banking before RT then artificial insemination is an option. A large tourniquet should be placed in plain sight at the client's bedside, in the event severe bleeding occurs. The purpose is to have the tourniquet available to apply to the residual limb to control bleeding if hemorrhaging was to occur.

A client who was in a motor vehicle collision was admitted to the hospital and the right knee was placed in skeletal traction. The nurse has documented this nursing diagnosis in the client's medical record: "Potential for impairment of skin integrity related to immobility from traction." Which nursing intervention is indicated based on this diagnosis statement? Release the traction q4h to provide skin care. Turn the client for back care while suspending traction. Provide back and skin care while maintaining the traction. Give back care after the client is released from traction.

Provide back and skin care while maintaining the traction. Maintaining skin integrity and providing back care is difficult when a client is in traction, but must be performed and is the correct intervention to maintain the client's skin integrity.

A client with type II diabetes arrives at the clinic with a blood glucose of 50 mg/dL. The nurse provides the client with 6 ounces of orange juice. In 15 minutes the client's capillary glucose is 74 mg/dL. What action should the nurse take? Obtain a specimen for serum glucose level. Administer insulin per sliding scale. Provide cheese and bread to eat. Collect a glycosylated hemoglobin specimen.

Provide cheese and bread to eat. Once blood glucose is greater than 70 mg/dL, the client should eat a regularly scheduled meal or a snack that contains protein and carbohydrates to help prevent hypoglycemia from recurring.

An adult client who is hospitalized after surgery reports sudden onset of chest pain and dyspnea. The client appears anxious, restless, and mildly cyanotic. The nurse should further assess the client for which condition? Pulmonary embolism. Heart failure. Tuberculosis. Bronchitis.

Pulmonary embolism. Post-surgical clients are at an increased risk for deep vein thrombosis (DVT), which may result in pulmonary embolism if the clot breaks off and travels to the lungs. Signs and symptoms of pulmonary embolism include chest pain, dyspnea, anxiety, restlessness, and - in severe cases - cyanosis.

The nurse is caring for a client who is two days postoperative. Which observation should alert the nurse to call the Rapid Response Team (RRT)? Fresh bleeding noted on abdominal surgical wound dressing. Pulse change from 85 to160 beats/minute lasting more than 10 minutes. Temperature of 103.1 F (39.5 C) and white blood cell (WBC) count of 16,000 mm3. Weakness, diaphoresis, reports of feeling faint. BP 100/56 mmHg.

Pulse change from 85 to160 beats/minute lasting more than 10 minutes. The RRT should be called to intervene for a postoperative client with an acute life-threatening change, such as a pulse change resulting in tachycardia for a prolonged time period.

The healthcare provider prescribes aluminum and magnesium hydroxide, 1 tablet PO PRN, for a client with chronic kidney disease (CKD) who is complaining of indigestion. Which intervention should the nurse implement? Administer 30 minutes before eating. Incorrect Evaluate the effectiveness 1 hour after administration. Instruct the client to swallow the tablet whole. Question the healthcare provider's prescription.

Question the healthcare provider's prescription. Magnesium agents are not usually used for clients with CKD due to the risk of hypermagnesemia, so this prescription should be questioned by the nurse.

A client with gastroesophageal reflux disease (GERD) has been experiencing severe reflux during sleep. Which recommendation by the nurse is most effective to assist the client? Losing weight. Decreasing caffeine intake. Avoiding large meals. Raising the head of the bed on blocks.

Raising the head of the bed on blocks. Raising the head of the bed on blocks (reverse Trendelenburg position) to reduce reflux and subsequent aspiration is the most non-pharmacological effective recommendation for a client experiencing severe gastroesophageal reflux during sleep.

The registered nurse (RN) is evaluating a client who presents with symptoms of viral gastroenteritis. Which assessment finding should the RN report to the healthcare provider? Dry mucous membranes and lips. Rebound abdominal tenderness over right lower quadrant. Dizziness when client ambulates from a sitting position. Poor skin turgor over client's wrist.

Rebound abdominal tenderness over right lower quadrant. Right lower quadrant (RLQ) rebound abdominal tenderness may be related to acute appendicitis and should be reported to the healthcare provider.

An adult client is admitted to the hospital burn unit with partial-thickness and full-thickness burns over 40% of the body surface area. In assessing the potential for skin regeneration, which should the nurse remember about full-thickness burns? Regenerative function of the skin is absent because the dermal layer has been destroyed. Tissue regeneration will begin several days following return of normal circulation. Debridement of eschar will delay the body's ability to regenerate normal tissue. Incorrect Normal tissue formation will be preceded by scar formation for the first year.

Regenerative function of the skin is absent because the dermal layer has been destroyed. Full-thickness burns destroy the entire dermal layer. Included in this destruction is the regenerative tissue. For this reason, tissue regeneration does not occur, and skin grafting is necessary.

After checking the urinary drainage system for kinks in the tubing, the nurse determines that a client who has returned from the post-anesthesia care has a dark, concentrated urinary output of 54 mL for the last 2 hours. Which priority nursing action should be implemented? Report the findings to the surgeon. Irrigate the indwelling urinary catheter. Apply manual pressure to the bladder. Increase the IV flow rate for 15 minutes.

Report the findings to the surgeon After surgery, an adult who weighs 132 pounds (60 kg) should produce about 60 mL of urine hourly (1 mL/kg/hour). Dark, concentrated, and low volume of urine output should be reported to the surgeon.

During CPR, when attempting to ventilate a client's lungs, the nurse notes that the chest is not moving. Which action should the nurse take first? Use a laryngoscope to check for a foreign body lodged in the esophagus. Reposition the head to validate that the head is in the proper position to open the airway. Turn the client to the side and administer three back blows. Perform a finger sweep of the mouth to remove any vomitus.

Reposition the head to validate that the head is in the proper position to open the airway. The most frequent cause of inadequate aeration of the client's lungs during CPR is the improper positioning of the head resulting in occlusion of the airway. The nurse should reposition the client's head and attempt to ventilate again, looking for the rise and fall of the chest.

Which assessment is most important for the nurse to perform on a client who is hospitalized for Guillain-Barre syndrome that is rapidly progressing? Respiratory effort. Unsteady gait. Intensity of pain. Ability to eat.

Respiratory effort. Guillain-Barre syndrome causes paralysis or weakness that typically starts at the feet and progresses upwards. As the condition progresses, the nurse must ensure that the client is able to breathe effectively.

The nurse is caring for a client with human immunodeficiency virus (HIV) infection who develops Mycobacterium avium complex (MAC). Which is the most significant desired outcome for this client? Free from injury of drug side effects. Return to pre-illness weight. Adequate oxygenation. Maintenance of intact perineal skin.

Return to pre-illness weight. MAC is an opportunistic infection that presents as a tuberculosis-like pulmonary process. MAC is a major contributing factor to the development of wasting syndrome, so the most significant desired outcome is the client's return to a pre-illness weight using oral, enteral, or parenteral supplementation as needed.

The nurse is preparing discharge instructions for a client who is going home with a surgical wound on the coccyx that is healing by secondary intention. What is the priority nursing diagnosis that should guide the discharge instruction plan? Acute pain. Risk for infection. Disturbed body image. Risk for deficient fluid volume.

Risk for infection. A wound healing by secondary intention is an open wound that is at risk for infection and the location of the wound near the anal area increases the risk for infection even more so.

A client with multiple sclerosis has experienced an exacerbation of symptoms, including paresthesias, diplopia, and nystagmus. Which instruction should the nurse provide? Stay out of direct sunlight. Restrict intake of high protein foods. Schedule extra rest periods. Go to the emergency room immediately.

Schedule extra rest periods. Exacerbations of the symptoms of MS occur most commonly as the result of fatigue and stress. The client should be encouraged to schedule extra rest periods to help reduce the symptoms.

The nurse is assessing a client's laboratory values following administration of chemotherapy. Which lab value leads the nurse to suspect that the client is experiencing tumor lysis syndrome (TLS)? Serum PTT of 10 seconds. Serum calcium of 5 mg/dL. Oxygen saturation of 90%. Hemoglobin of 10 g/dL.

Serum calcium of 5 mg/dL. Tumor lysis symdrome (TLS) results in hyperkalemia, hypocalcemia, hyperuricemia, and hyperphosphatemia. A serum calcium level of 5, which is low, is an indicator of possible tumor lysis syndrome.

A 46-year-old female client is admitted for acute renal failure secondary to diabetes and hypertension. Which test is the best indicator of adequate glomerular filtration? Serum creatinine. Blood Urea Nitrogen (BUN). Sedimentation rate. Urine specific gravity.

Serum creatinine Creatinine is a product of muscle metabolism that is filtered by the glomerulus, and blood levels of this substance are not affected by dietary or fluid intake. An elevated creatinine strongly indicates nephron loss, reducing filtration.

Three weeks after discharge for an acute myocardial infarction (MI), a client returns to the cardiac center for follow-up. When the nurse asks about sleep patterns, the client states that sleep is fine but that the spouse has moved into the spare bedroom to sleep after returning home. The client states, "I guess we will never have sex again after this." Which response is best for the nurse to provide? Sexual intercourse can be strenuous on the heart, but closeness and intimacy, such as holding and cuddling, can be maintained with your spouse. Sexual activity can be resumed whenever you and your spouse feel like it because the sexual response is more emotional than physical. Discuss questions about sexual activity with your healthcare provider because sexual activity may be limited by your heart damage. Sexual activity is similar in cardiac workload and energy expenditure as climbing two flights of stairs and may be resumed like other activities.

Sexual activity is similar in cardiac workload and energy expenditure as climbing two flights of stairs and may be resumed like other activities. Sexual intercourse after an MI has been found to require no more energy expenditure or cardiac stress than walking briskly up two flights of stairs, so if the client does not experience shortness of breath or chest discomfort doing the stairs then it should be okay to resume sexual activity.

The nurse working in a postoperative surgical clinic is assessing a woman who had a left radical mastectomy for breast cancer. Which factor puts this client at greatest risk for developing lymphedema? She sustained an insect bite to her left arm yesterday. She has lost twenty pounds since the surgery. Her healthcare provider now prescribes a calcium channel blocker for hypertension. Her hobby is playing classical music on the piano.

She sustained an insect bite to her left arm yesterday. A radical mastectomy interrupts lymph flow, and the increased lymph flow that occurs in response to the insect bite increases the risk for the occurrence of lymphedema.

The nurse working on a telemetry unit finds a client unconscious and in pulseless ventricular tachycardia (VT). The client has an implanted automatic defibrillator. Which action should the nurse implement? Prepare the client for transcutaneous pacemaker. Shock the client with 200 joules per hospital policy. Use a magnet to deactivate the implanted pacemaker. Observe the monitor until the onset of ventricular fibrillation.

Shock the client with 200 joules per hospital policy. A client with an automatic defibrillator who is experiencing pulseless ventricular tachycardia (VT) must be externally shocked with 200 joules per hospital policy to restore an effective cardiac rhythm. The automatic defibrillator is obviously malfunctioning.

The nurse is assessing a client who has a history of Parkinson's disease for the past 5 years. Which symptoms would this client most likely exhibit? Loss of short-term memory, facial tics and grimaces, and constant writhing movements. Shuffling gait, masklike facial expression, and tremors of the head. Extreme muscular weakness, easy fatigability, and ptosis. Numbness of the extremities, loss of balance, and visual disturbances.

Shuffling gait, masklike facial expression, and tremors of the head. Parkinson's Disease is one of the most common neurologic progressive disorders of the older client. Shuffling gait, masklike facial expression, and tremors of the head and hands are common clinical features of Parkinsonism.

Which finding should the nurse report to the healthcare provider for a client with a circumferential extremity burn? Full thickness burns rather than partial thickness. Supinates extremity but unable to fully pronate the extremity. Slow capillary refill in the digits with absent distal pulse points. Inability to distinguish sharp versus dull sensations in the extremity.

Slow capillary refill in the digits with absent distal pulse points. A circumferential burn can form an eschar that results from burn exudate fluid that dries and acts as a tourniquet as fluid shifts occur in the interstitial tissue. As edema increases tissue pressure, blood flow to the distal extremity is compromised, which is manifested by slow capillary refill and absent distal pulses, so the healthcare provider should be notified about any compromised circulation that requires escharotomy.

In preparing a discharge plan for a 22-year-old male client diagnosed with Buerger's disease (thromboangiitis obliterans), which referral is most important? Genetic counseling. Twelve-step recovery program. Clinical nutritionist. Smoking cessation program.

Smoking cessation program. Buerger's disease is strongly related to smoking or the use of some other form of tobacco which affects the circulation in the arms and legs leading to infection and gangrene and sometimes amputation of the affected area. The most effective means of controlling symptoms and disease progression is through smoking cessation. The cause of Buerger's disease is unknown; a genetic predisposition is possible, but unproven.

The nurse is caring for a client who has been diagnosed with primary hyperaldosteronism. Which laboratory test result should the nurse expect an increase in the serum level? Sodium. Antidiuretic hormone. Potassium. Glucose.

Sodium Clients with primary aldosteronism exhibit an increase in serum sodium levels (hypernatremia) and have profound decline in the serum levels of potassium (hypokalemia)--hypertension is the most prominent and universal sign. Antidiuretic hormone is decreased with diabetes insipidus. Glucose is not affected by primary aldosteronism.

The registered nurse (RN) is assessing a client who was discharged home after management of chronic hypertension. Which equipment should the RN instruct the client to use at home? Exercise bicycle. Sphygmomanometer. Blood glucose monitor. Weekly medication box.

Sphygmomanometer Self-awareness is the best way for a client to manage chronic hypertension, so the client should obtain a sphygmomanometer and learn how to monitor blood pressure daily and maintain a record.

A female client receiving IV vasopressin for esophageal varice rupture reports to the nurse that she feels substernal tightness and pressure across her chest. Which PRN protocol should the nurse initiate? Start an IV nitroglycerin infusion. Nasogastric lavage with cool saline. Increase the vasopressin infusion. Prepare for endotracheal intubation.

Start an IV nitroglycerin infusion. Vasopressin is used to promote vasoconstriction, thereby reducing bleeding from the esophageal varice. Vasoconstriction of the coronary arteries can lead to angina and myocardial infarction, and should be counteracted by IV nitroglycerin per prescribed protocol.

Which description of symptoms is characteristic of a client diagnosed with trigeminal neuralgia (tic douloureux)? Tinnitus, vertigo, and hearing difficulties Sudden, stabbing, severe pain over the lip and chin. Facial weakness and paralysis. Difficulty in chewing, talking, and swallowing.

Sudden, stabbing, severe pain over the lip and chin. Trigeminal neuralgia is characterized by paroxysms of pain, similar to an electric shock, in the area innervated by one or more branches of the trigeminal nerve (5th cranial). Women are more often afflicted with this condition and generally occurs in clients over the age of 50 years old.

The nurse is preparing a teaching plan for a client who is newly diagnosed with Type 1 diabetes mellitus. Which clinical cues should the nurse describe when teaching the client about hypoglycemia? Sweating, trembling, tachycardia. Polyuria, polydipsia, polyphagia. Nausea, vomiting, anorexia. Fruity breath, tachypnea, chest pain.

Sweating, trembling, tachycardia. Sweating, dizziness, and trembling are signs of hypoglycemic reactions related to the release of epinephrine as a compensatory response to the low blood sugar.

A 77-year-old female client is admitted to the hospital. She is confused, has no appetite, is nauseated and vomiting, and is complaining of a headache. Her pulse rate is 43 beats per minute. It is most important for the nurse to assess for which finding? Wearing dentures. Use of aspirin prior to admit. Prescribed nitroglycerin for chest pain. Takes digitalis.

Takes digitalis. Although it is important to obtain a complete medication history, the symptoms described are classic for digitalis toxicity, and assessment of this problem should be made promptly. Elderly persons are particularly susceptible to digitalis intoxication which manifests itself in such symptoms as anorexia, nausea, vomiting, diarrhea, headache, and fatigue.

The nurse formulates the nursing problem of urinary retention related to sensorimotor deficit for a client with multiple sclerosis. Which nursing intervention should the nurse implement? Teach the client techniques of intermittent self-catheterization. Decrease fluid intake to prevent over distention of the bladder. Use incontinence briefs to maintain hygiene with urinary dribbling. Explain that anticholinergic drugs will decrease muscle spasticity.

Teach the client techniques of intermittent self-catheterization. Bladder control is a common problem for clients diagnosed with multiple sclerosis. A client with urinary retention should receive instructions about self-catheterization to prevent bladder distention.

A client asks the nurse about the purpose of beginning chemotherapy (CT) because the tumor is still very small. Which information supports the explanation that the nurse should provide? Side effects are less likely if therapy is started early. Collateral circulation increases as the tumor grows. Sensitivity of cancer cells to CT is based on cell cycle rate. The cell count of the tumor reduces by half with each dose.

The cell count of the tumor reduces by half with each dose. Initiating chemotherapy while the tumor is small provides a better chance of eradicating all cancer cells because 50% of cancer cells or tumor cells are killed with each dose.

A male client with a prostatic stent is preparing for discharge. Which information is most important for the nurse to provide the client prior to discharge? Ongoing antibiotic therapy is needed for one year. The client should not undergo magnetic resonance imaging. Increased frequency of assessment for prostatic cancer is needed. The client should not be catheterized through the stent for at least three months.

The client should not be catheterized through the stent for at least three months. A prostatic stent is a cylinder shape tube that is placed in the urethra to relieve prostatic pressure from an enlarged prostate and improve urine flow. To prevent complications, the client should be cautioned against catheterization through the prostatic stent for three months after stent placement.

The registered nurse (RN) is caring for a client with tuberculosis (TB) who is taking a combination drug regimen. The client complains about taking "so many pills." What information should the RN provide to the client about the prescribed treatement? The development of resistant strains of TB are decreased with a combination of drugs. Compliance to the medication regimen is challenging but should be maintained. Side effects are minimized with the use of a single medication but is less effective. The treatment time is decreased from 6 months to 3 months with this standard regimen.

The development of resistant strains of TB are decreased with a combination of drugs. Combination therapy is necessary to decrease the development of resistant strains of TB and ensure treatment efficacy.

A 58-year-old client, who has no health problems, asks the nurse about the Pneumovax vaccine. The nurse's response to the client should be based on which information? The vaccine is given annually before the flu season to those over 50 years of age. The immunization is administered once to older adults or persons with a history of chronic illness. The vaccine is for all ages and is given primarily to those persons traveling overseas to areas of infection. The vaccine will prevent the occurrence of pneumococcal pneumonia for up to five years.

The immunization is administered once to older adults or persons with a history of chronic illness. It is recommended by the CDC (Dec 2016) that persons over 65 years of age and those with a history of chronic illness receive the vaccine once in a lifetime.

A Korean-American client, who speaks very little English, is being discharged following surgery. Which nurse should the nurse manager assign to provide the discharge instructions for the client? A graduate registered nurse (RN) with three weeks of experience. The registered nurse (RN) case-manager for the unit with 1 year's experience. A "floating" registered nurse (RN) with five years of nursing experience. An Korean-American practical nurse (PN) with six years of nursing experience.

The registered nurse (RN) case-manager for the unit with 1 year's experience. The RN case-manager is the best qualified nurse to assess and provide discharge educational needs, obtain resources for the client, enhance coordination of care, and prevent fragmentation of care.

A client with early breast cancer receives the results of a breast biopsy and asks the nurse to explain the meaning of staging and the type of receptors found on the cancer cells. Which explanation should the nurse provide? Lymph node involvement is not significant. Small tumors are aggressive and indicate poor prognosis. The tumor's estrogen receptor guides treatment options. Stage I indicates metastasis.

The tumor's estrogen receptor guides treatment options. Treatment decisions and prediction of prognosis are related to the tumor's receptor status, such as estrogen and progesterone receptor status which commonly are well-differentiated, have a lower chance of recurrence, and are receptive to hormonal therapy. Tumor staging designates tumor size and spread of breast cancer cells into axillary lymph nodes, which is one of the most important prognostic factors in early-stage breast cancer.

The nurse is planning preoperative teaching for a client who will undergo a radical neck dissection and total laryngectomy. Which information has the greatest priority for this client? Prognosis after treatment is excellent. Techniques for esophageal speech are relatively easy to learn with practice. The stoma should never be covered after this type of surgery. There is a radical change in appearance as a result of this surgery.

There is a radical change in appearance as a result of this surgery. Radical neck dissection is the removal of lymphatic drainage channels and nodes, sternocleidomastoid muscle, spinal accessory nerve, jugular vein, and submandibular area. The overall outcome of this type of surgery causes the neck to be disfigured, so the radical change in appearance, "Alteration in body image" will be a priority in the care of this client.

During lung assessment, the nurse places a stethoscope on a client's chest and instructs him/her to say "99" each time the chest is touched with the stethoscope. Which would be the correct interpretation if the nurse hears the spoken words "99" very clearly through the stethoscope? This is a normal auscultatory finding. May indicate pneumothorax. May indicate pneumonia. Correct May indicate severe emphysema

This is a normal auscultatory finding. This test (whispered pectoriloquy) demonstrates hyperresonance and helps determine the clarity with which spoken words are heard upon auscultation. Normally, the spoken word is not well transmitted through lung tissue, and is heard as a muffled or unclear transmission of the spoken word. Increased clarity of a spoken word is indicative of some sort of consolidation process (e. g., tumor, pneumonia), and is not a normal finding.

A client with Meniere's disease is incapacitated by vertigo and is lying in bed, grasping the side rails, and staring at the television. Which nursing intervention should the nurse implement? Encourage fluids to 3000 mL per day. Change the client's position every two hours. Keep the head of the bed elevated 30 degrees. Turn off the television and darken the room.

Turn off the television and darken the room. To decrease the client's vertigo during an acute attack of Meniere's disease, any visual stimulant or rotational movement, such as sudden head movements or position changes, should be minimized. To effectively manage the client's symptoms, darken the room by minimizing fluorescent lights, flickering television lights, and distracting sounds.

The nurse is teaching a female client about the best time to plan sexual intercourse in order to conceive. Which information should the nurse provide? Two weeks before menstruation. Vaginal mucous discharge is thick. Low basal temperature. First thing in the morning.

Two weeks before menstruation. Ovulation typically occurs 14 days before menstruation begins during a typical 28 day cycle. Sexual intercourse should occur within 24 hours of ovulation for an increase chance of conception to occur. High estrogen levels occur during ovulation and increase the vaginal mucous membrane characteristics to become more "slippery" and stretchy, along with a rise in basal temperature. The timing during the day is not as significant in determining conception as the day before and after ovulation.

The registered nurse (RN) is assessing a male client who arrives at the clinic with severe abdominal cramping, pain, tenesmus, and dehydration. The RN discovers that the client has had 14 to 20 loose stools with rectal bleeding. When taking the client's medical history, which information is most for the nurse to obtain? Irritable bowel syndrome. Diverticulitis. Crohn's disease. Ulcerative colitis.

Ulcerative colitis. The RN should ask the client if he has a history of ulcerative colitis, which is characterized by severe abdominal cramping, pain, tenesmus, and dehydration.

The nurse is assessing a client with a chest tube that is attached to suction and a closed drainage system. Which finding is most important for the nurse to further assess? Upper chest subcutaneous emphysema. Tidaling (fluctuation) of fluid in the water-seal chamber. Constant air bubbling in the suction-control chamber. Pain rated "8" (0-10) at the insertion site.

Upper chest subcutaneous emphysema. Subcutaneous emphysema is a complication and indicates air is leaking beneath the skin surrounding the chest tube.

The registered nurse (RN) is caring for a client who developed oliguria and was diagnosed with sepsis and dehydration 48 hours ago. Which assessment finding indicates to the RN that the client is stabilizing? Urine output of 40 mL/hour. Apical pulse 100 and blood pressure 76/42. Urine specific gravity 1.001. Tented skin on dorsal surface of hands.

Urine output of 40 mL/hour. A decrease in urinary output is a sign of dehydration. When the urine output returns to a normal range, 40 mL/hour, the client's kidneys are perfusing adequately and indicates the client's status is stablizing.

What is the priority nursing action while caring for a client on a ventilator when an electrical fire occurs in the intensive care unit? Tell another staff member to bring extinguishing equipment to the bedside. Close the doors to the client's area when attempting to extinguish the fire. Use a bag-valve-mask resuscitator while removing the client from the area. Implement an emergency protocol to remove the client from the ventilator.

Use a bag-valve-mask resuscitator while removing the client from the area. A client on a ventilator should have respirations maintained with a manual bag-valve-mask resuscitator while being moved away from the oxygen wall outlet and fire source.

Which intervention should the nurse implement that best confirms placement of an endotracheal tube (ETT)? Use an end-tital CO2 detector. Auscultate for bilateral breath sounds. Obtain pulse oximeter reading. Check symmetrical chest movement.

Use an end-tital CO2 detector. The end-tidal carbon dioxide detector indicates the prescence of CO2tidal by a color change or a number indicated on the detector, which is supporting evidence that the ETT is in the trachea, not the esophagus.

Which discharge instruction is most important for a client after a kidney transplant? Weigh weekly. Report symptoms of secondary Candidiasis. Use daily reminders to take immunosuppressants. Stop cigarette smoking.

Use daily reminders to take immunosuppressants. After a renal transplantation, acute rejection is a high risk for several months. The organ recipient will have to take immunosuppressive therapy for the rest of their lives, such as corticosteroids and azathioprine, to prevent organ transplant rejection. Discharge instructions include measures such as daily reminders to ensure the client takes these medications regularly to prevent organ rejection from occurring.

Which method elicits the most accurate information during a physical assessment of an older client? Ask the client to recount one's health history. Obtain the client's information from a caregiver. Review the past medical record for medications. Use reliable assessment tools for older adults.

Use reliable assessment tools for older adults. Specific assessment tools designed for an older adult consider age-related physiologic and psychosocial changes related to aging and provide the most accurate and complete information. Examples of these tools include the Older Adult Resource Services Center Instrument (OARS), mini-mental assessment, fall risk, depression (Geriatric Depression Scale), and skin breakdown risk (Braden Scale).

A client who is sexually active with several partners requests an intrauterine device (IUD) as a contraceptive method. Which information should the nurse provide? Using an IUD offers no protection against sexually transmitted diseases (STD), which increase the risk for pelvic inflammatory disease (PID). Getting pregnant while using an IUD is common and is not the best contraceptive choice. Relying on an IUD may be a safer choice for monogamous partners, but a barrier method provides a better option in preventing STD transmission. Selecting a contraceptive device should consider choosing a successful method used in the past.

Using an IUD offers no protection against sexually transmitted diseases (STD), which increase the risk for pelvic inflammatory disease (PID) The use of an intrauterine device (IUD) provides the client with no protection from sexually transmitted diseases (STD).

The nurse assesses a long-term resident of a nursing home and finds the client has a fungal infection (candidiasis) beneath both breasts. To prevent nosocomial infection, which protocol should the nurse review with the rest of the staff? Follow contact isolation procedures. Wash hands after caring for the client. Wear gloves when providing personal care. Restrict pregnant staff or visitors into the room.

Wash hands after caring for the client. The organism Candida albicans, that causes this infection, is part of the normal flora on the skin of most adults. Good handwashing is all that is needed to prevent nosocomial spread.

Healthcare workers must protect themselves against becoming infected with HIV. The Center for Disease Control has issued guidelines for healthcare workers in relation to protection from HIV. These guidelines include which recommendation? Place HIV positive clients in strict isolation and limit visitors. Wear gloves when coming in contact with the blood or body fluids of any client. Conduct mandatory HIV testing of those who work with AIDS clients. Freeze HIV blood specimens at -70 F to kill the virus.

Wear gloves when coming in contact with the blood or body fluids of any client. The CDC guidelines recommend that healthcare workers use gloves when coming in contact with blood or body fluids from any client since HIV is infectious before the client becomes aware of their exposure and/or symptomatic.

Which assessment finding should the nurse identify that indicates a client with an acute asthma exacerbation is beginning to improve after treatment? Wheezing becomes louder. Cough remains unproductive. Vesicular breath sounds decrease. Bronchodilators stimulate coughing

Wheezing becomes louder. In an acute asthma attack, air flow may be so significantly restricted that breath sounds and wheezing are diminished. If the client is successfully responding to bronchodilators and respiratory treatments, wheezing should become louder as the air flow increases in the airways. As the airways open and mucous is mobilized in response to treatment, the cough should become more productive.

The nurse is planning care for a client with newly diagnosed diabetes mellitus that requires insulin. Which assessment should the nurse identify before beginning the teaching session? Present knowledge related to the skill of injection. Intelligence and developmental level of the client. Willingness of the client to learn the injection sites. Financial resources available for the equipment.

Willingness of the client to learn the injection sites. If a client is incapable or does not want to learn, it is unlikely that learning will occur, so motivation is the first factor the nurse should assess before teaching.

Which information about mammograms is most important to provide a post-menopausal female client? Breast self-examinations are not needed if annual mammograms are obtained. Radiation exposure is minimized by shielding the abdomen with a lead-lined apron. Yearly mammograms should be done regardless of previous normal x-rays. Women at high risk should have annual routine and ultrasound mammograms.

Yearly mammograms should be done regardless of previous normal x-rays. There are different recommendations from different agnecies. For a client with no risk factors, the earliest breast screening recommendation is a yearly mammogram at the age 40 and till the age of 54. After that every two years.

The nurse is completing the health assessment of a 79-year-old client who denies any significant health problems. Which finding requires the most immediate follow-up assessment? Kyphosis with a reduction in height. Dilated superficial veins on both legs. External hemorrhoids with itching. Yellowish discoloration of the sclerae.

Yellowish discoloration of the sclerae. In a geriatric client, a yellowish discoloration (jaundice) of the sclerae is not a normal finding and may indicate liver damage and requires further assessment.

The registered nurse (RN) assesses arterial blood gas results of a client that has emphysema. Which finding is consistent with respiratory acidosis? pH 7.32, pCO 2 46 mmHg, HCO 3 24 MEq/L. pH 7.45 , pCO 2 37 mmHg, HCO 3 24 mEq/L. pH 7.34, pCO 2 36 mmHg, HCO 3 21 mEq/L. pH 7.46, pCO 2 35 mmHg, HCO 3 28 mEq/L.

pH 7.32, pCO 2 46 mmHg, HCO 3 24 MEq/L. Normal ABG ranges are pH 7.35 to 7.45; pCO2 35 to 45 mmHg; HCO3 21 to 28 mEq/L, and pO2 80 to 100 mmHg. An ABG of pH 7.32, pCO2 46 mmHg, HCO3 24 MEq/L represents a client with respiratory acidosis which is characterized by: low pH, pCO2 higher than normal, and HCO3 within normal limits.

The client is taking digoxin for congestive heart failure. The nurse would be correct in withholding a dose of digoxin based on which assessment? serum digoxin level is 1.5. blood pressure is 104/68. serum potassium level is 3. apical pulse is 68/min.

serum potassium level is 3. Hypokalemia can precipitate digitalis toxicity in persons receiving digoxin which will increase the chance of dangerous dysrhythmias (normal potassium level is 3.5 to 5.5 mEq/L).

A client has taken steroids for 12 years to help manage chronic obstructive pulmonary disease (COPD). When making a home visit, which nursing assessment is of greatest importance to this client? pulse rate, both apically and radially. blood pressure, both standing and sitting. temperature. skin color and turgor.

temperature Long term use of steroids by COPD clients is effective in suppressing inflammation in their airways making it easier for them to breath, but at the same time suppresses the immune system, placing the client at risk for infection, so it is very important to obtain the client's temperature.


संबंधित स्टडी सेट्स

Biology 104 Dr. Carr Ole Miss Test 2

View Set

Chapter 17 CTS 220 (N02) Adv Hard/Software Support

View Set

Consent, Capacity, and Competency Issues

View Set

Medical Surgical Nurse Certification

View Set

Eco 2411 Money and Banking Midterm Exam

View Set

Electronic Resources and Health/Information Literacy

View Set

Module 1 Earth Space Science practice's

View Set